New Pedia

Download as pdf or txt
Download as pdf or txt
You are on page 1of 37

EXAM QUESTIONS OF “CHILD DISEASES 2 (FACULTY PEDIATRICS)”

FOR STUDENTS OF 3TH YEAR (VI SEMESTER) 2023-2024 a.y.

1. Deficiency of vitamin A in children causes:


a) Goiter d) Increased risk of mortality
b) Poor cognitive development с) Beri-beri
c) Poor bone growth

2. Severe acute malnutrition in young children is defined as:


a) Weight-for-age Z score <-3
b) Height-for-age Z score <-3 and edema
c) Height-for-age Z score <-3 or weight-for-height Z score <-3 or edema
d) Height-for-age Z score <-3 or weight-for-age Z score <-3 or edema
e) Weight-for-age Z score <+3

3. Deficiency of which of the following nutrients can lead to anemia?


a) Iodine and vitamin C d) Vitamin D and zinc
b) Copper and iron e) Vitamin C and iron
c) Zinc and protein

4. For which groups of people does the World Health Organization recommend zinc supplements?
a) Pregnant and lactating women
b) Children with severe malnutrition or diarrhea
c) All children in low-income areas with a high prevalence of stunting
d) Older people with low incomes
e) Children with anemia

5. Vitamin D deficiency can often be found as a deficiency of a single nutrient, that is, in a well-nourished person.
The main reason why this may happen is the following:
a) Foods containing high amounts of vitamin D are eaten by both poor and rich people depending on various cultural
reasons
b) The vitamin D content of foods depends on the soil in which the food was grown, so is not related to wealth or
age
c) Most vitamin D in the body is not obtained from food, so access to food is not an important factor in determining
vitamin D status
d) Diets containing sufficient amounts of other nutrients to promote growth may increase vitamin D requirements
to the point of becoming limiting.
e) Foods containing large amounts of vitamin D are eaten by rich people for various cultural reasons

6. The mother of the 4-month's child has addressed to the doctor with complaints on a decrease of appetite,
regurgitation, subferile temperature in her child. It is known, that from the 2nd month of life the child receives vitamin
D3 in preventive dose /500 IU each day/, from the 3-rd month of life because of irritability and sweating was prescribed
general ultra-violet insolation. Objectively: large fontanelle is closed, skin with perioral cyanosis. Laboratory
investigations: Sulcovich test is positive (++), serum calcium – 3.5 mmol/l. Name the most probable diagnosis:
a) Hypervitaminosis D d) Hypovitaminosis D
b) Spasmophylia e) Avitaminosis D
c) Personal intolerance of vitamin D

7. Which features are important in the differential diagnosis of biliary tract dyskinesia and cholecystitis?
a) Pain in the right upper quadrant d) Enlargement of the liver
b) Symptoms vascular dystonia e) Enlargement of the liver and spleen
c) Disease duration is 2 years

8. A girl, 12 years, complains of paroxysmal pain in the right upper quadrant, which increases during exercise, after
eating fatty foods, nausea, loss of appetite, headache, fatigue. Sick for 5 years. These objective examination: skin pale
tongue with yellow-gray stratification. Abdomen soft, painful in the right upper quadrant, positive symptoms Ortner,
Murphy, Kera, the liver acts from the edge of costal arch to 2 cm edge sharp.The tendency to constipation.These
ultrasound: thickened wall and sealed intrahepatic bile ducts and gall bladder. What is the main diagnosis you set sick?
a) Acute cholecystitis. d) Biliary dyskinesia hypertensive type
b) Chronic cholecystocholangitis e) Chronic cholecystitis
c) Biliary dyskinesia hypokinetic type
9. Identify the most important sign of B12-deficiency anemia.
a) Hyperchromic anemia d) Anemia, lymphopenia, monocytopenia
b) Hypochromic anemia e) Leukemia
c) Thrombocytopenia

10. The mother complains of lack of teeth and lower limb deformation in a child 8 months. The baby was born
weighing 3000 grams healthy from birth on artificial feeding, prevention of rickets received. During the inspection,
the overall condition is not violated, pale skin, teeth no large fontanel has a size of 3 x 3 cm lower aperture chest
expanded palpable ribs rosary. There O-curvature of the lower extremities. With the weakening of the heart listen
colors, functional systolic murmur. What is your diagnosis?
a) Rickets d) The vitamin D-dependent rickets
b) Congenital brittle bones e) Tubulopathy
c) Hondrodystrofiya

11. Forms of chronic nutritional disorders in children are:


a) Hypotrophy, Hypostature, Paratrophy d) Hypotrophy, Hypostature, hypertrophy
b) Dystrophy, Hypostature, Paratrophy e) Dystrophy, Hypostature, hyportrophy
c) Dystrophy, Hypostature, hypertrophy

12. Boy, 14 years old, 3 years periodically complains of pain in the epigastric region on an empty stomach, nausea
and heartburn. During fibrogastroduodenoscopy gastroduodenitis and the signs of ulcerative mucosal defect
duodenum. What eradication complex medications most effective in the treatment of peptic ulcer?
a) Tetracycline + + trihopol d) Clarithromycin + amoxicillin + trihopol
b) Amoxicillin + + trihopol + thiamin e) Clarithromycin + amoxicillin + de nol
c) Trihopol+ de nol + + papaverine.

13. Infants with PI (ponderal index) of 0.9 – 1.1 and NI (nutritional index) of 1.0 – 0.9 are considered:
a) Atrophic d) Hypotrophic
b) Eutrophic e) Hypotrophic and hypertrophic
c) Hypertrophic

14. A positive Murphy's sign is highly indicative of acute cholecystitis. Which of the following statements
demonstrates this maneuver?
1. Pain during inhalation as the examiner places his/her hand over the rt. upper quadrant
2. Pain during exhalation as the examiner places his/her hand over the rt. upper quadrant
3. Pain in rt. lower quadrant on palpation of the lt. lower quadrant
4. Pain during hyper-extension of the rt. hip joint
5. Pain on palpation over Burney's point

15. In the children's department taken the girl under the age of 8 years. He complains of frequent attacks of pain in
the right upper quadrant and near the belly button that occurs after physical activity over the last 3 months, tendency
to constipation. During the physical examination, skin color natural, soft abdomen, pain in the right upper quadrant,
the liver acts from the edge of costal arch to 2 cm elastic. When duodenal sounding portions could not be obtained, a
portion of A - unchanged. Worsening of the disease is most likely the patient?
a) Chronic cholecystocholangitis.
b) Biliary dyskinesia hypertensive type.
c) Biliary dyskinesia hypokinetic-hypertonic by type.
d) Biliary dyskinesia on-hypertensive hiperkinetych-it type
e) Chronic cholecystitis

16. Iron deficiency anemia is characterized by:


a) Low serum glucose d) Low level of enzymes
b) Low serum ferritin e) Low serum Iron
c) High level of enzymes

17. Hospital pneumonia is one that developed:


a) during 6 hours being in the hospital d) during 48 hours being in the hospital
b) during 12 hours being in the hospital e) during 72 hours being in the hospital
c) during a 24 hours being in the hospital
18. A newborn baby presented to the emergency with complaint of vomiting since birth. X-ray abdomen revealed
double bubble. The diagnosis is:
a) Duodenal atresia c) Gastric outlet obstruction
b) Esophageal atresia d) Hirschsprung Disease
e) Gastric atresia

19. A neonate presents with cyanosis which fails to improve with inhaled oxygen. ECG shows left axis deviation.
The diagnosis is:
a) Hypoplastic Left Heart Syndrome d) Tricuspid Atresia
b) Mitral stenosis e) Aortic stenosis
c) Tetrology of Fallots

20. The causes of death in protein energy malnutrition:


a) Anemic heart failure d) Congenital heart failure
b) Constipation e) Loose stool
c) Bleeding

21. In the biochemical analysis of blood сhild with malnutrition will be:
a) Hyperglycemia d) Hypoproteinemia
b) Hypercalcemia e) Hypoproteinemia and hypercalcemia
c) Hyperalbuminemia

22. The chest X-ray sign typical for acute pneumonia is:
a) Strengthening of pulmonary picture (lung pattern)
b) Infiltrative shadows
c) Emphysema
d) Dilation of lungs' roots
e) Thickening and deformation of the bronchovascular pattern of the lungs

23. In most cases an acute pneumonia at children of the early age develops as a result of:
a) Overheat d) Upper respiratory tract viral infection
b) Super cooling e) Upper respiratory tract bacterial infection
c) Violation of the regime

24. To the criteria of an acute pneumonia diagnosis belong:


a) Lymphocytes in complete blood count d) Respiratory acidosis
b) Epidemiological history e) Respiratory alkalosis
c) Changes in nasal ways

25. At moderate pneumonia antibacterial therapy lasts:


a) 5 - 7 days c) 10 - 14 day e) 3 - 5 days
b) 7 - 10 days d) 14 - 20 days
26. The etiological agents of hospital pneumonia are:
a) E. coli, proteus d) Staphylacoccus, influenza
b) Brucella, streptococcus e) Staphylacoccus, E. coli
c) Pneumococcus, tuberculosis

27. The beginning of obstructive bronchitis is:


a) Catarrhal syndrome d) Expiration dyspnea
b) Allergic reactions e) Cough, temperature
c) Inspiration dyspnea

28. Which main clinical features are useful in the diagnosis of bronchiolitis:
a) Paroxysmal cough d) Dyspnea
b) Wheezing e) Inspiration dyspnea
c) Tachypnea
29. Bronchitis is caused most often by:
a) Fungi d) Parasites
b) Viruses e) Allergens
c) Bacteria

30. What is the most common congenital heart defect with a left to right shunt causing congestive heart failure in the
pediatric age group?
a) Atrial septal defect
b) Atrioventricular canal d)Patent ductus arteriosus
c)Ventricular septal defect e) Tetrology of Fallots
31. The main symptom of (PDA) patent ductus arteriosus in children is:
a) Tachycardia or other arrhythmia c) Continuous machine-like murmur
b) Shortness of breath and other respiratory d) Enlarged heart
problems e) Cough

32. Which heart murmur is characteristic for (PDA) patent ductus arteriosus in children?
a) Systolic d)
b) Diastolic e) Continuous murmur
c) Systolic and diastolic f) Systolic and diastolic

33. What is the criterion of carditis in acute rheumatic fever ARF in children?
a) Damage of myocardium and endocardium d) Damage of pericardium
b) Damage of cardialgias e) Damage of pericardium and endocardium
c) Damage of epicardium

34. Common causes of mortality in primary nephrotic syndrome is/are:


a) Acute renal failure d) Seizure
b) Congestive heart failure e) Carditis
c) Peritonitis

35. What is the most common congenital heart defect with a left to right shunt causing congestive heart failure in the
pediatric age group?
a) Atrial septal defect d) Patent ductus arteriosus
b) Atrioventricular canal e) Tetrology of Fallots
c) Ventricular septal defect

36. What is the main difference between Rheumatic Heart Disease (RHD) and Acute Rheumatic Fever (ARF)?
a) In ARF there is an elevated ESR
b) In RHD there is a prolonged P-R interval d) In RHD there is evidence of chronic heart disease
c) In ARF there is a history of arthralgias e) In ARF there is a history of carditis

37. The three D's of pellagra are:


a) diarrhea, dementia, dermatitis c) detoxicaton, dizziness,deafness
b) diuresis, dilation of bladder, dementia d) demineralization, dementia, dizziness
e) dementia, dizziness, dermatitis

38. Child 10 years old complaine on loss of appetite, heartburn, pain in the navel, which is dull and occurs more often
within 2-3 hours after eating and in the morning - on an empty stomach. The pain decreases after a meal. Ну is sick
for three years. OBJECTIVELY: Skin is pale. Abdomen is soft, pain during deep palpation in the epigastric and
pyloroduodenal areas. Positive Mendel symptom. What is the most probable diagnosis?
a) Chronic cholecystocholangitis d) Peptic ulcer
b) Crohn's Disease e) Chronic gastritis
c) Chronic gastroduodenitis

39. What is the main criterion of acute rheumatic fever (ARF) in children?
a) Carditis d) Pneumonia
b) Hepatitis e) Dermatitis
c) Nephritis

40. Paratrofia in children is manifested by:


a) Decreased body weight proportionally to the length
b) Increased body weight proportionally to the length
c) Increased body weight in stunted growth
d) Decreased body weight with an accelerated growth
e) Decreased body weight with an accelerated length

41. For giardiasis, all the symptoms are characteristic, except:


a) Loss of appetite d) Children have a delay in physical development
b) Itching e) Nausea
c) Rumbling and bloating
42. With peptic ulcer of the stomach, "early pain" appears after taking food through:
a) 15-30 min d) 5 min
b) 60-70 minutes e) 1 min
c) 60-120 min

43. The patient, 15 years old, 5 years suffering from duodenal ulcer. After exercise felt severe pain in the epigastric
region and right upper quadrant. It was vomiting. There is a weak pulse, sharp pain from the navel to the right, the
disappearance of hepatic dullness during percussion. What caused condition that requires emergency care?
a) Spontaneous pneumothorax. d) Exacerbations of cholelithiasis
b) Duodenal ulcer perforation. e) Ulcer bleeding
c) Aggravation of kidney stones (renal colic).

44. A girl, 12 years, complains of paroxysmal pain in the right upper quadrant, which increases during exercise, after
eating fatty foods, nausea, loss of appetite, headache, fatigue. Sick for 5 years. These objective examination: skin pale
tongue with yellow-gray stratification. Abdomen soft, painful in the right upper quadrant, positive symptoms Ortner,
Murphy, Kera, the liver acts from the edge of costal arch to 2 cm edge sharp. The tendency to constipation. These
ultrasound: thickened wall and sealed intrahepatic bile ducts and gall bladder. What is the main diagnosis you set sick?
a) Acute cholecystitis. d) Biliary dyskinesia hypokinetic by type
b) Chronic cholecystitis. e) Biliary dyskinesia hypertensive type
c) Chronic cholecystocholangitis.

45. In the 12-year-old boy there is aching pain in the epigastrium after 1,5 hours after eating and fasting, periodic
nausea, vomiting, heartburn. Recently, the child gets tired quickly, complaining of headache, weakness. Chronic antral
gastritis was diagnosed due to results of clinical and additional methods of examinations. What clinical syndromes
are the leading?
a) Pain, dyspepsial, asthenic-vegetative d) Dyspepsial, ossalgic, intoxication
b) Asthenic, pain, cephalgic e) Pain, intoxication, intoxication
c) Pain, intoxication, asthenic

46. The patient, 15 years old, 5 years suffering from duodenal ulcer. After exercise felt severe pain in the epigastric
region and right upper quadrant. It was vomiting. There is a weak pulse, sharp pain from the navel to the right, the
disappearance of hepatic dullness during percussion. What caused condition that requires emergency care?
a) Duodenal ulcer perforation. d) Exacerbations of cholelithiasis
b) Spontaneous pneumothorax e) Ulcer bleeding
c) Aggravation of kidney stones (renal colic)

47. A 10 years old boy has acute glomerulonephritis during a mouth. He has edema. In urine: protein – 2,5 g/l, in the
biochemical blood test: total protein – 48 g/l, cholesterol- 9,8 mmol|l. What from this medicine must be appointed to
the child in the complex of pathogenetical therapy?
a) Delagyl d) Heparin
b) Plaquenyl e) Curantil
c) Prednisolon

48. The boy of 3 has an edema syndrome like as anasarca. Blood pressure - 95/60. In the general analysis of urine:
protein – 6,3 g\l, leucocytes 2-3 in field of view, red corpuscles 1-2 in field of view, cylinders - 2-3 in field of view.
General protein of blood – 44,2 g\l, albumen – 38,1%, cholesterol of blood – 8,6 mmol\l. What clinical variant of
acute glomerulonephritis does take place probably?
a) Acute glomerulonephritis with nephritic syndrome
b) Acute glomerulonephritis with nefrotic syndrome
c) Acute glomerulonephritis with isolated urine syndrome
d) Acute glomerulonephritis with nefrotic syndrome, hematuria and arterial hypertension
e) Acute glomerulonephritis with hematuria

49. A 12-year-old boy is registered at a dispensary with a cardiorheumatologist with a diagnosis of rheumatism,
active phase, rheumacarditis with mitral valve damage. How long should secondary bicillin-drug prevention of
rheumatism be carried out?
a) for life d) 18 years old
b) 3 years e) 25 years
c) 1 year
50. Pneumonia during auscultation is characterized by:
a) The presence of dry wheezing, which increases on exhalation, wheezing breathing
b) The presence of crepitation or small-bubbly wet wheezing, weakened or bronchial breathing
c) The presence of dry wheezing, scattered throughout the pulmonary fields, hard breathing
d) The presence of small bubbly wheezes at the height of inspiration, various wet and dry wheezes, which change the
character after coughing
e) The presence of wet and dry wheezing

51. A 8-year-old child has been ill for three years. Complaints of abdominal pain, more in the right hypochondrium,
subfebrility. A positive symptom of Kera. In blood tests, an increase in ESR is noted. What disease causes such a
clinical picture:
a) Chronic enterocolitis d) Biliary dyskinesia
b) Chronic cholecystitis e) Acute intestinal infection
c) Chronic duodenitis

52. A 1.5-year-old child was admitted to the hospital on the 3rd day of the disease. He became acutely ill, the
temperature was 39.2 C, weakness, coughing, refusal to eat, shortness of breath appeared from the 3rd day, cough
intensified. Upon admission: adynamic, pale, cyanosis of the nasolabial triangle, temperature - 38.5 C, respiratory
rate - 52 per minute. Above the lungs - shortening of the sound in the right scapular region. On the R-gram:
homogeneous infiltration of segments 8-10 on the right. Make a diagnosis.
a) Obstructive bronchitis d) Bronchiolitis
b) Bronchitis e) Segmental pneumonia
c) Interstitial pneumonia

53. Identify the following diagnoses is most probable. A boy 12 years old admitted to the hospital with intermittent
high fever, allergic rash, pain and swelling in the knee and ankle joints, increase of peripheral lymph nodes, liver
and spleen. In blood test - leukocytes 27x109/l, ESR - 65mm/hour, increased immunoglobulin M and G.
a) Sepsis d) Rheumatc fever
b) Systemic lupus erythematosus e) Leukemia
c) Systemic juvenile rheumatoid arthritis

54. Put a preliminary diagnosis. Patient 14yrs old complained of intense pain in the right lumbar region, chills,
accompanied by fever up to 39°C. The abdomen is soft, painful in the right area. Palpation of right kidney is
painful. In the blood: leukocytes 30.0 x109/L, ESR - 50 mm/hour. In urine an.: acid reaction, leukocytes in the
entire field of vision. According to the US - the left kidney is normal, the contours of the right kidney are
increased.
a) Right paranephritis d) Swelling of the right kidney
b) Acute right-sided pyelonephritis e) Polycystic kidney degeneration
c) Tuberculosis of the right kidney

55. Find the disease that led to the current state of the patient.13 years old girl was hospitalized with straining pain
in the left hypochondrium, which irradiates to the back. He notes nausea, decreased appetite, weight loss, vomiting
without relief, diarrhea. He has been ill for over 5 years. Exacerbation has developed because of errors in the diet.
Objective: t ° = 37,0°C, pulse rate 94 per minute, BP 125/75. Skin is pale, pain in the epigastrium, right and left
hypochondrium. In the blood test: Leuk. 10.4 x 109/l, ESR 22 mm/hour.
a) Stomach ulcer d) Chronic pancreatitis
b) Chronic gastritis e) Chronic enterocolitis
c) Chronic cholecystitis

56. Indicate the drug which should be prescribed primarily for treatment. 6 years old boy complains of an acute
abdominal pain, which arises after mental loading, use of cold drinks, ice-cream. The diagnosis: Dyskinesia of
gallbladder, hypertonic type.
a) Spasmolitics and choleretics d) Antioxidants
b) Sedative and cholikinetics e) Antibiotics
c) Choleretics and cholikinetics

57. Which of the following is the treatment of choice for peptic ulcer disease caused by H. pylori infection?
a) PPI, amoxicillin, and clarithromycin d) PPI and erythromycin
b) H2 blockers e) PPI and azithromycin
c) Erythromycin
58. A 7-year-old child present with hypoalbuminemia, edema,hyperlipidemia and proteinuria.The edema is in the
periorbital region initially and eventually spreads to the rest of the body. The patient is given steroid therapy
and the disease goes away.What is a key morphological feature of the patient’s disease?
a) Fusion of the foot processes d) Hemosiderin laden macrophages in the kidney
b) Destruction of the basement membrane e) None of the above
c) Destruction of the glomerulus

59. Against the background of acute respiratory viral infections, an 11-year-old child developed a dry cough. During
auscultation, hard breathing, dry and single medium-bubbly diffuse wheezing on inspiration, decreasing after
coughing, is determined. Radiologically symmetrical enhancement of the pulmonary pattern in the basal zones. In the
general blood test – relative lymphocytosis. Specify the probable diagnosis.
a) Acute obstructive bronchitis d) Pneumonia
b) Acute simple bronchitis e) Recurrent bronchitis
c) Bronchiolitis

60. A 1.5-year-old child was admitted to the hospital on the 3rd day of the disease. He became acutely ill, the
temperature was 39.2 C, weakness, coughing, refusal to eat, shortness of breath appeared from the 3rd day, cough
intensified. Upon admission: adynamic, pale, cyanosis of the nasolabial triangle, temperature - 38.5 C, respiratory rate
- 52 per minute. Above the lungs - shortening of the sound in the right scapular region. On the R-gram: homogeneous
infiltration of segments 8-10 on the right. Make a diagnosis.
a) Obstructive bronchitis d) Bronchiolitis
b) Bronchitis e) Segmental pneumonia
c) Interstitial pneumonia

61. Which of the following is the best next step after 8 weeks of proton pump inhibitor trial without improvement
of chronic epigastric discomfort?
a) Colonoscopy d) Endoscopy with biopsy
b) H2 inhibitors e) Abdominal ultrasonography
c) Continue proton pump inhibitors

62. What is an effective test for detection of H. pylori infection?


a) Ultrasound scan d) Urine analysis
b) Urea breath e) GFR (glomerular filtration rate)
c) Blood picture

63. Which of the following medical conditions is the most common cause of dyspepsia in children?
a. Functional dyspepsia d. Pancreatitis
b. Gastric cancer e. Esophageal cancer
c. Peptic ulcer disease

64. Which of the following is the best next step in the management of Functional dyspepsia with alarm features in
children?
1. Proton pump inhibitors 4. Diet and lifestyle modification
2. Testing for Helicobacter pylori 5. H2 antagonists
3. Endoscopy

65. What types of medication are considered for a client with peptic ulcer disease (PUD) in children?
1. H2 receptor antagonists, antibiotics, proton pump inhibitors, probiotics
2. H2 receptor antagonists, antibiotics, proton pump inhibitors, mucosal protectants
3. H2 receptor antagonists, antibiotics, probiotics, antacids
4. H2 receptor antagonists, nonsteroidal anti-inflammatory drugs, proton pump inhibitors, mucosal protectants
5. Nonsteroidal anti-inflammatory drugs, antibiotics, proton pump inhibitors, mucosal protectants, probiotics

66. Which of the following best describes the flow of bile from the liver to the gallbladder and then the small
intestines?
1. Right and left hepatic duct - Common hepatic duct - Cystic duct - Gallbladder - Cystic duct - Common bile
duct - Sphincter of Oddi
2. Right and left hepatic duct - Common bile duct - Cystic duct - Gallbladder - Cystic duct - Common hepatic
duct - Sphincter of Oddi
3. Sphincter of Oddi - Right and left hepatic duct - Common hepatic duct - Cystic duct - Gallbladder - Cystic duct
- Common bile duct
4. Right and left hepatic duct - Common bile duct - Gallbladder - Cystic duct - Common hepatic duct - Sphincter
of Oddi
5. Right and left hepatic duct - Gallbladder - Common bile duct - Cystic duct - Liver - Cystic duct - Common
hepatic duct - Sphincter of Oddi

67. A positive Murphy's sign is highly indicative of acute cholecystitis. Which of the following statements
demonstrates this maneuver?
6. Pain during inhalation as the examiner places his/her hand over the rt. upper quadrant
7. Pain during exhalation as the examiner places his/her hand over the rt. upper quadrant
8. Pain in rt. lower quadrant on palpation of the lt. lower quadrant
9. Pain during hyper-extension of the rt. hip joint
10. Pain on palpation over McBurney's point

68. A patient has pain on the right side of the abdomen radiating to the right shoulder. The serum transaminases,
alkaline phosphatase, and serum lipase are increased. Which location is the MOST likely site of impaction of the
stone?
1. The stone is present at the sphincter of Oddi. 4. The stone is impacted at the right hepatic duct.
2. The stone is impacted at the cystic duct. 5. The stone is impacted at the neck of the
3. The stone is impacted at the left hepatic bile gallbladder.
duct.

69. Which of the following is a characteristic gallblabber abnormality?


1. Phrygian cap 4. Accessory bile duct
2. Absence of entire extrahepatic duct system 5. Choledochal cyst
3. Atresia of bile duct

70. Which of the following is the treatment of choice for symptomatic cholelithiasis?
1. Cholecystectomy 4. Ursodeoxycholic acid
2. Observation 5. Cholecystokinetic drug
3. Sphincterotomy

71. A 16-year-old girl presents with right upper quadrant pain. She has a BMI of 32 kg/m2. She complains of
feeling nauseated at times. What is the probable type of gallbladder stone causing the right upper quadrant pain?
1. Pure cholesterol stone 4. Pigment stone
2. Mixed stones 5. Uric acid stone
3. Combined stones

72. What respiratory rate might indicate that an infant has severe bronchiolitis in children?
1. < 50 breaths/min 4. 30–40 breaths/min
2. 70 breaths/min 5. 30 breaths/min
3. 40 breaths/min

73. What kind of supports can help infants manage viral bronchiolitis in children?
1. Antipyretics, nasal suction, hydration, supplemental oxygen
2. Antibiotics, nasal suction, hydration, stopping breastfeeding
3. Antipyretics, nasal suction, fluid restriction, supplemental oxygen
4. Antibiotics, nasal suction, hydration, supplemental oxygen
5. Antipyretics, stopping breastfeeding, supplemental oxygen

74. What is the age of a typical patient presenting with bronchiolitis in children?
1. Under 5 years of age 4. Between 1-2 years of age
2. Over 3 years of age 5. Between 2-3 years of age
3. Under 2 years of age

75. What are risk factors that contribute to bronchiolitis in children? Select all that apply.
1. Being premature, attending daycare, being breastfed
2. Being premature, attending daycare, having heart or lung defects
3. Being premature, being the first child, having heart or lung defects
4. Being breastfed, attending daycare, having heart or lung defects
5. Older age, attending daycare, having heart or lung defects
76. Which of the following options provides the correct order of lung structures from proximal to distal?
1. Terminal bronchioles, respiratory bronchioles, alveolar sacs, alveolar ducts
2. Respiratory bronchioles, terminal bronchioles, alveolar ducts, alveolar sacs
3. Respiratory bronchioles, terminal bronchioles, alveolar sacs, alveolar ducts
4. Terminal bronchioles, respiratory bronchioles, alveolar ducts, alveolar sacs
5. Terminal bronchioles, alveolar ducts, respiratory bronchioles, alveolar sacs

77. 16-year-old male presents with a history of productive cough and low grade fever. He was previously healthy
and does not suffer from any chronic ailments. He is diagnosed with tracheobronchitis. What is the next step in
management?
1. Supportive care only 4. Oral third generation cephalosporins
2. Inhaled steroids 5. Oral neuraminidase inhibitors
3. Inhaled bronchodilators

78. Which factors would indicate a diagnosis of bronchitis in children?


1. History of acute onset of persistent cough for 1–3 weeks, no clinical signs of pneumonia (fever, rales, tachypnea),
an infiltrate on chest radiograph
2. History of acute onset of persistent cough for 1–3 weeks, no clinical signs of pneumonia (fever, rales, tachypnea),
chest X-ray to rule out pneumonia
3. History of acute onset of persistent cough for 1–3 weeks, no clinical signs of pneumonia (fever, rales, tachypnea),
a pleural effusion or empyema on chest radiograph
4. History of acute onset of persistent cough for 1–3 weeks, no clinical signs of pneumonia (fever, rales, tachypnea),
an elevated WBC count, in the range of 15,000-40,000/mm3
5. History of acute onset of persistent cough for 1–3 weeks, clinical signs of pneumonia (fever, rales, tachypnea),
positive blood culture

79. A patient diagnosed with pneumonia was hospitalized for 10 days. In spite of an initial improvement with
administration of amoxicillin/clavulanic acid and clarithromycin, by day 6 the patient's condition began to deteriorate.
Ticarcillin was added with good response, and eventual resolution of symptoms. What was the probable causative
organism in this patient?
1. Streptococcus pneumoniae 4. Mycoplasma pneumoniae
2. Pseudomonas aeruginosa 5. Haemophilus influenza
3. Legionella pneumophila

80. Which of the following best defines hospital-acquired pneumonia in children?


1. Pneumonia acquired in the community requiring hospitalization
2. Pneumonia developed by healthcare personnel
3. Pneumonia associated with foreign body aspiration
4. Pneumonia acquired 48 hours after a hospital admission unrelated to pneumonia
5. Pneumonia associated with medication

81. Which of the following symptoms represent atypical pneumonia caused by Mycoplasma pneumonia in
children?
1. Nonproductive cough, rhonchi, and myalgia
2. Nonproductive cough, wheezing, and myalgia
3. Productive cough, rhonchi, and arthralgia
4. Nonproductive cough, rhonchi, and arthralgia
5. Productive cough, wheezing, and arthralgia

82. Which of the following is the most common cause of atypical pneumonia in children?
1. L. pneumophila 4. Respiratory syncytial virus
2. M. pneumoniae 5. S. aureus
3. C. pneumoniae

83. What is the indication for doing a blood culture in patients with pneumonia in children?
1. Routine work-up for pneumonia 4. Non-productive cough
2. Presence of hemoptysis 5. Suspicion of sepsis
3. Negative chest x-ray findings

84. Which of the following is a common cause of acute pneumonia, especially in the pediatric population?
1. Mycoplasma pneumonia 4. Tuberculosis
2. Nocardia 5. Aspergillus
3. Actinomycosis

85. What age range of children should be treated with amoxicillin for bacterial pneumonia?
1. Children 5–8 years old 3. Children 5–10 years old
2. Children < 5 years old 4. Children > 5 years old
5. Adolescents
86. Which of the following is the antibiotic of choice for bacterial pneumonia in an otherwise healthy child 1-4
years of age?
1. Amoxicillin 4. Moxifloxacin
2. Azithromycin 5. Vancomycin
3. Clindamycin

87. Which of the following is the bacterial pathogen most commonly found in cases of community acquired
pneumonia?
1. Streptococcus pneumoniae 4. Staphylococcus aureus
2. Group A Streptococci 5. Staphylococcus epidermidis
3. Group B Streptococci

88. Which sign or symptom would suggest a primary viral pneumonia due to influenza over a typical influenza
illness?
1. Dyspnea 4. Productive cough
2. Consolidative sounds on auscultation 5. Significant myalgias
3. Fever

89. Which of the following drugs is added for the treatment of infants with pneumonia due to Chlamydia
trachomatis?
1. Ciprofloxacin 4. Amoxicillin
2. Azithromycin 5. Moxifloxacin
3. Clindamycin

90. What types of pathogens typically cause pneumonia in children 0 to 18 years old?
1. Mycoplasma pneumoniae, Haemophilus influenzae, Chlamydia pneumoniae, Adenoviruses
2. Streptococcus pneumoniae, Haemophilus influenzae, Chlamydia pneumoniae, Influenza virus A and B
3. Mycoplasma pneumoniae, Haemophilus influenzae, Chlamydia pneumoniae, Escherichia coli
4. Streptococcus pneumoniae, Mycoplasma pneumoniae, Haemophilus influenzae, Chlamydia pneumoniae
5. Staphylococcus aureus, Haemophilus influenzae, Chlamydia pneumoniae, Influenza virus A and B

91. Which of the following test has a rapid response time and is reliable in providing the diagnosis of mycoplasma
pneumonia?
1. Polymerase chain reaction (PCR) 4. Chest x-ray
2. Mycoplasma antibody rise in titres 5. Gram stain on sputum sample
3. Sputum culture

92. What assessments are part of a full respiratory examination?


1. Inspection, palpation, percussion, blood pressure
2. Inspection, palpation, percussion, auscultation
3. Inspection, pulse volume, percussion, auscultation
4. Inspection, palpation the liver, percussion, auscultation
5. Inspection, palpation, retraction, auscultation

93. Which neonatal infection is caused by Chlamydia trachomatis 2–3 weeks after birth?
1. Asthma 4. Pneumonia
2. Emphysema 5. Bronchiectasis
3. Bronchitis

94. Which of the following cardiac diseases is commonly caused by acute rheumatic fever?
1. Myocardial infarction 4. Mitral valve prolapse
2. Infectious endocarditis 5. Pulmonic stenosis
3. Mitral stenosis

95. In acute rheumatic fever, antibodies against group A streptococcus cross-react with antigens present on which
of the following structures of the heart?
1. Epicardium and myocardium 4. Epicardium and valves
2. Myocardium and valves 5. Endothelium and epithelium
3. Epicardium and endocardium
96. Which of the following is most likely the main indication for antibiotic treatment in children with group A
Streptococcus pharyngitis?
1. Reducing symptom severity 3. Reducing symptom duration
2. Prevention of post-streptococcal 4. Prevention of acute rheumatic fever
glomerulonephritis 5. Prevention of spread to others

97. Which of the following is a major criterion used to diagnose acute rheumatic fever?
1. Carditis 4. Sore throat
2. Erythema multiforme 5. Hepatitis
3. Sjogren’s syndrome

98. Which of the following is a major criterion used to diagnose acute rheumatic fever?
1. Erythema multiforme 4. Sore throat
2. Polyarthritis 5. Prolonged P-R interval
3. Sjogren’s syndrome

99. Which of the following is a major criterion used to diagnose acute rheumatic fever?
1. Erythema marginatum 4. Sore throat
2. Erythema multiforme 5. Prolonged P-R interval
3. Fever

100. Which of the following is a major criterion used to diagnose acute rheumatic fever?
1. Erythema multiforme 4. Prolonged P-R interval
2. Fever 5. Subcutaneous nodules
3. Sore throat

101. Which of the following is a major criterion used to diagnose acute rheumatic fever?
1. Erythema multiforme 4. Chorea
2. Fever 5. Prolonged P-R interval
3. Sore throat

102. Which of the following is a minor criterion used to diagnose acute rheumatic fever?
1. Fever 4. Sore throat
2. Chorea 5. Subcutaneous nodules
3. Erythema multiforme

103. Which of the following is a minor criterion used to diagnose acute rheumatic fever?
1. Prolonged P-R interval 4. Sore throat
2. Chorea 5. Subcutaneous nodules
3. Erythema multiforme

104. Which of the following is a minor criterion used to diagnose acute rheumatic fever?
1. Chorea 4. Arthralgia
2. Erythema multiforme 5. Subcutaneous nodules
3. Sore throat

105. Which of the following is a minor criterion used to diagnose acute rheumatic fever?
1. Elevated acute-phase reactants (erythrocyte sedimentation rate; C-reactive protein)
2. Supporting evidence of antecedent group A streptococcal infection
3. Carditis
4. Polyarthritis
5. Subcutaneous nodules

106. A 12-year-old boy has sore throat, fever, tender and swollen anterior cervical nodes, and bilateral tonsillar
exudates for 2 days. He has no coryza, conjunctivitis, cough, hoarseness, anterior stomatitis, discrete ulcerative
lesions or vesicles, or diarrhea. What is the next best step in management?
1. Initiate antibiotics 4. Prescribe acetaminophen
2. Obtain blood cultures 5. Refer for hospitalization
3. Obtain a rapid streptococcal antigen test

107. What is the underlying pathophysiology of mitral valve damage in acute rheumatic fever?
1. Cross-reactive antibodies targeting valve tissue
2. Congenital mucinous infiltration of valve leaflets
3. Bacterial colonization of abnormal valve tissue
4. Infectious endocarditis
5. Reactivation of latent infectious organisms

108. Which of the following valvular abnormalities is least likely to be a direct result of acute rheumatic heart
disease?
1. Pulmonary stenosis 4. Aortic stenosis
2. Mitral stenosis 5. Aortic regurgitation
3. Mitral regurgitation

109. Which of the following describes the murmur of mitral stenosis?


1. Opening snap followed by a rumbling mitral diastolic murmur.
2. Mid-systolic click with a mid-diastolic rumble
3. Systolic crescendo-decrescendo murmur
4. Early diastolic decrescendo murmur
5. High-pitched holosystolic murmur

110. Which of the following is true regarding mitral stenosis?


1. Early-diastolic murmur, difficulty in closing
2. Diastolic murmur, difficulty in opening
3. Holosystolic murmur, difficulty in opening
4. Late systolic murmur, difficulty in opening
5. Crescendo-decrescendo, floppy valve

111. Which of the following is the most common cause of mitral stenosis?
1. Dystrophic calcification 4. Rheumatic heart disease
2. Bicuspid valve 5. Intravenous drug use
3. Congenital mitral stenosis

112. Which part of the heart is often enlarged due to mitral stenosis?
1. Left atrium 4. Aorta
2. Left ventricle 5. Right ventricle
3. Right atrium

113. How would you differentiate mitral regurgitation from mitral stenosis in a gross pathologic specimen of the
heart?
1. The left atrium and left ventricle are enlarged in mitral regurgitation.
2. The left atrium is atrophied in mitral stenosis.
3. The left ventricle is hypertrophic in mitral stenosis.
4. The left atrium is atrophic in mitral regurgitation.
5. The interventricular septum is enlarged in mitral stenosis

114. Aortic stenosis directly affects which of the given chambers of the heart?
1. Left ventricle 4. Atriums
2. Right ventricle 5. Right atrium
3. Left atrium

115. Which one of the listed conditions leads to left to right shunting?
1. Pulmonary stenosis 4. Coarctation of the aorta
2. Atherosclerotic heart disease 5. Aortic stenosis
3. Patent ductus arteriosus

116. Which one of the listed conditions leads to left to right shunting?
1. Atrial septal defect 4. Coarctation of the aorta
2. Pulmonary stenosis 5. Aortic stenosis
3. Atherosclerotic heart disease

117. Which of the following is the most common type of atrial septal defect?
1. Sinus venosus 4. Coronary sinus
2. Ostium secundum 5. Atrioventricular canal defect
3. Ostium primum

118. Classic continuous machinery murmur is typically heard in which of the following conditions?
1. Aortic stenosis 4. Coarctation of aorta
2. Ventricular septal defect 5. Atrial septal defect
3. Patent ductus arteriosus

119. Ventricular septal defect has which of the following types of murmurs?
1. Pansystolic murmur 4. Early diastolic murmur
2. Still’s murmur 5. Ejection systolic murmur
3. Mid diastolic murmur
120. Which one of the following is considered a congenital cyanotic heart disease?
1. Tetralogy of Fallot 4. Pericardial effusion
2. Coarctation of the aorta 5. Cardiomegaly
3. Cardiomyopathy

121. Which of the following allows blood to bypass the pulmonary circulation during fetal life?
1. Ventricular septum 4. Atrioventricular canal
2. Ductus arteriosus 5. Truncus arteriosus
3. Atrial septum

122. Which of the following congenital heart lesions is most likely to require surgical attention in early childhood?
1. Atrial septal defect 4. Transposition of the great arteries leading to
2. Patent ductus arteriosus blue baby
3. Pulmonic stenosis 5. Ventricular septal defect

123. Which of the following congenital heart lesions results in persistent blood flow directly from the aorta into the
pulmonary artery?
1. Patent ductus arteriosus 4. Coarctation of the aorta
2. Atrial septal defect 5. Pulmonic stenosis
3. Ventricular septal defect

124. What condition may result from a long-standing, left-to-right shunting of blood, which involves reversal into a
cyanotic right-to-left shunt?
1. Eisenmenger's syndrome 4. Mitral stenosis
2. Aortic stenosis 5. Tricuspid stenosis
3. Pulmonic stenosis

125. Which of the given heart diseases mainly causes obstruction to blood flow from the left ventricle?
1. Truncus arteriosus 4. Aortic stenosis
2. Patent ductus arteriosus 5. Transposition of great arteries
3. Atrial septal defect

126. Which of the following conditions is most likely to present with cyanosis?
1. Transposition of the great arteries 4. Coarctation of the aorta
2. Atrial septal defect 5. Bicuspid aortic valve
3. Ventricular septal defect

127. What is the most common congenital heart defect?


1. Atrial septal defect 4. Ventricular septal defect
2. Patent foramen ovale 5. Tricuspid atresia
3. Mitral atresia

128. What heart defect prevents blood flow from the right atrium to the right ventricle?
1. Tricuspid atresia 4. Transposition of the great arteries
2. Mitral atresia 5. Atrial septal defect
3. Ventricular septal defect

129. What is the classical radiologic finding in tetralogy of Fallot?


1. Boot-shaped heart 4. Scimitar sign
2. Egg-on-a-string sign 5. Gooseneck sign
3. Snowman sign

130. In coarctation of the aorta, what would be the expected quality of the pulses and blood pressure?
1. Notably decreased in vessels before point of coarctation
2. Notably increased in vessels below the coarctation
3. Notably decreased in vessels below the coarctation
4. Unaffected by the coarctation
5. Notably increased only in collateral circulation

131. Which of the following medications should be administered to keep the patent ductus arteriosus open?
1. Indomethacin 4. Atropine
2. Prednisolone 5. Prostaglandin E1
3. Montelukast

132. The oxygenated blood from the placenta is delivered to the fetus via what vessel?
1. Umbilical vein 4. Anterior cardinal vein
2. Vitelline vein 5. Posterior cardinal vein
3. Umbilical artery

133. What is the first medication you give a baby with suspected ductal dependent cyanotic heart disease?
1. Epinephrine drip 4. Indomethacin
2. Bicarb infusion 5. Prostaglandins
3. Surfactant

134. Which of the following is a common finding in aortic coarctation?


1. Hypertension 4. Right ventricular hypertrophy
2. Left atrial hypertrophy 5. Tachycardia
3. Patent ductus arteriosus

135. Which of the following is characteristic of aortic coarctation?


1. Diminished radial pulses 4. Diminished femoral pulses
2. Increased ankle brachial index 5. Bifid carotid pulse
3. Narrow pulse pressure

136. Which of the following is a more accurate description of murmur associated with aortic stenosis?
1. A systolic ejection murmur heard best at the apex
2. A crescendo-decrescendo diastolic murmur heard best at the apex
3. A systolic ejection murmur heard best at the base of the heart
4. A crescendo-decrescendo diastolic murmur heard best at the base of the heart
5. A constant murmur heard in both systole and diastole

137. Which of the following features will allow a baby with transposition of the great vessels to remain alive at
birth?
1. Patent ductus arteriosus 4. An arteriovenous malformation in the lungs
2. Mitral valve stenosis allowing for shunting
3. Pulmonary atresia 5. Surfactant administration at birth

138. A 17-year-old student and football player is referred to your primary care office for evaluation of high blood
pressure. During an annual check-up, he was noted to have an arterial blood pressure of 155/90 mm Hg without
presenting symptoms. On physical examination, his blood pressure is 152/94 mm Hg, and his heart rate is 59
beats/min. On auscultation, a systolic ejection murmur is heard over the spine between the shoulder blades. Of note
is that his femoral pulses are not well felt. His laboratory test results are normal; however, an ECG indicates left
ventricular hypertrophy. Which of the following is the most likely diagnosis?
1. Coarctation of the aorta 4. Hypertrophic obstructive cardiomyopathy
2. Aortic stenosis 5. Patent ductus arteriosus
3. Bicuspid aortic valve

139. Which of the following cardiac diseases is acyanotic congenital heart lesion causing a volume load?
1. Coarctation of the aorta 4. Atrial septal defect
2. Tetralogy of Fallot 5. Aortic stenosis
3. Transposition of the great arteries

140. Which of the following cardiac diseases is acyanotic congenital heart lesion causing a volume load?
1. Coarctation of the aorta 4. Ventricular septal defect
2. Tetralogy of Fallot 5. Aortic stenosis
3. Transposition of the great arteries

141. Which of the following cardiac diseases is acyanotic congenital heart lesion causing a pressure load?
1. Coarctation of the aorta 4. Aortic stenosis
2. Tetralogy of Fallot 5. Patent ductus arteriosus
3. Transposition of the great arteries

142. Which of the following is a diagnostic criterion for acute kidney injury (AKI)?
1. Persistent evidence of kidney damage or a decrease in function for at least 3 months
2. Hematuria with dysmorphic red blood cells
3. Oliguria (urine volume ≤ 0.5 mL/kg/hr for 6 hr)
4. Heavy proteinuria (protein excretion greater than 3.5 g/24 hours)
5. Multiple cysts on renal imaging

143. Which of the following conditions is characterised by an abrupt loss of kidney function leading to a rapid
decline in the glomerular filtration rate (GFR), accumulation of waste products such as blood urea nitrogen (BUN)
and creatinine, and dysregulation of extracellular volume and electrolyte homeostasis?
1. Acute kidney injury 4. Vesicoureteral reflux
2. Chronic kidney disease 5. Renal stones
3. Pyelonephritis

144. Which of the following patients would be diagnosed with stage G1 chronic kidney disease?
1. Patient B: irreversible kidney damage; GFR 65 mL/min per 1.73 m2)
2. Patient C: irreversible kidney damage; GFR 50 mL/min per 1.73 m2)
3. Patient D: irreversible kidney damage; GFR 44 mL/min per 1.73 m2)
4. Patient A: irreversible kidney damage; GFR 95 mL/min per 1.73 m2)
5. Patient E: irreversible kidney damage; GFR 14 mL/min per 1.73 m2)

145. Which of the following patients would be diagnosed with stage G2 chronic kidney disease?
1. Patient A: irreversible kidney damage; GFR 65 mL/min per 1.73 m2)
2. Patient B: irreversible kidney damage; GFR 95 mL/min per 1.73 m2)
3. Patient C: irreversible kidney damage; GFR 50 mL/min per 1.73 m2)
4. Patient D: irreversible kidney damage; GFR 44 mL/min per 1.73 m2)
5. Patient E: irreversible kidney damage; GFR 14 mL/min per 1.73 m2)

146. Which of the following patients would be diagnosed with stage G3a chronic kidney disease?
1. Patient B: irreversible kidney damage; GFR 95 mL/min per 1.73 m2)
2. Patient A: irreversible kidney damage; GFR 50 mL/min per 1.73 m2)
3. Patient C: irreversible kidney damage; GFR 65 mL/min per 1.73 m2)
4. Patient D: irreversible kidney damage; GFR 44 mL/min per 1.73 m2)
5. Patient E: irreversible kidney damage; GFR 14 mL/min per 1.73 m2)

147. Which of the following is a diagnostic criterion for acute kidney injury (AKI)?
1. Persistent evidence of kidney damage or a decrease in function for at least 3 months
2. Hematuria with dysmorphic red blood cells
3. Heavy proteinuria (protein excretion greater than 3.5 g/24 hours)
4. Multiple cysts on renal imaging
5. Increase in serum creatinine by ≥ 0.3 mg/dL from baseline within 48 hr

148. Which of the following is a diagnostic criterion for acute kidney injury (AKI)?
1. Persistent evidence of kidney damage or a decrease in function for at least 3 months
2. Hematuria with dysmorphic red blood cells
3. Heavy proteinuria (protein excretion greater than 3.5 g/24 hours)
4. Increase in serum creatinine to ≥ 1.5 times baseline within the prior 7 days
5. Multiple cysts on renal imaging

149. Which of the following conditions is characterised by lower urinary tract symptoms (dysuria, urgency, new-
onset urge incontinence, frequency, lower abdominal pain) with no fever or low-grade fever (<38 0C) together with
a significant growth of bacteria on urine culture?
1. Acute cystitis 4. Vesicoureteral reflux
2. Chronic kidney disease 5. Renal stones
3. Acute pyelonephritis

150. Which of the following conditions is characterised by fever with or without abdominal pain, loin pain together
with a significant growth of bacteria (usually a single organism) on urine culture?
1. Acute pyelonephritis 2. Acute cystitis
3. Chronic kidney disease 5. Renal stones
4. Vesicoureteral reflux

151. Which of the following are the common causes of prerenal acute kidney injury?
1. Glomerulonephritis: postinfectious; lupus erythematosus; membranoproliferative
2. Hemolytic-uremic syndrome, acute tubular necrosis, cortical necrosis
3. Dehydration, sepsis, cardiac failure
4. Acute interstitial nephritis, tumor lysis syndrome, toxin and drugs
5. Posterior urethral valves, ureteropelvic junction obstruction, urolithiasis

152. Which of the following are the common causes of postrenal acute kidney injury?
1. Posterior urethral valves, ureteropelvic junction obstruction, urolithiasis
2. Dehydration, sepsis, cardiac failure
3. Glomerulonephritis: postinfectious; lupus erythematosus; membranoproliferative
4. Hemolytic-uremic syndrome, acute tubular necrosis, cortical necrosis
5. Acute interstitial nephritis, tumor lysis syndrome, toxin and drugs

153. Which of the following are the common causes of intrinsic renal acute kidney injury?
1. Posterior urethral valves, ureteropelvic junction obstruction, urolithiasis
2. Dehydration, sepsis, cardiac failure
3. Urethral strictures, Hemorrhagic cystitis, Neurogenic bladder
4. Glomerulonephritis: postinfectious; lupus erythematosus; membranoproliferative
5. Gastroenteritis, Hemorrhage, Burns, Anaphylaxis

154. Which of the following are the common causes of acute tubular necrosis (ATN) in critically ill infants and
children?
1. Systemic lupus erythematosus
2. Severe and prolonged ischemic/hypoxic injury and nephrotoxic insult
3. Posterior urethral valves, ureteropelvic junction obstruction, urolithiasis
4. Urethral strictures, Hemorrhagic cystitis, Neurogenic bladder
5. Acute poststreptococcal glomerulonephritis

155. What is vesicoureteral reflux grade 2?


1. Reflux only fills the ureter without dilation.
2. Reflux fills and mildly dilates the ureter and the collecting system.
3. Reflux fills and grossly dilates the ureter and the collecting system with blunting of the calices.
4. Reflux fills the ureter and the collecting system without dilation.
5. Massive reflux grossly dilates the collecting system. All the calices are blunted with a loss of papillary
impression, and intrarenal reflux may be present. There is significant ureteral dilation and tortuosity.

156. What is vesicoureteral reflux grade 3?


1. Reflux fills and mildly dilates the ureter and the collecting system.
2. Reflux only fills the ureter without dilation.
3. Reflux fills the ureter and the collecting system without dilation.
4. Reflux fills and grossly dilates the ureter and the collecting system with blunting of the calices.
5. Massive reflux grossly dilates the collecting system. All the calices are blunted with a loss of papillary
impression, and intrarenal reflux may be present. There is significant ureteral dilation and tortuosity.

157. What is vesicoureteral reflux grade 4?


1. Reflux only fills the ureter without dilation.
2. Reflux fills and grossly dilates the ureter and the collecting system with blunting of the calices.
3. Reflux fills the ureter and the collecting system without dilation.
4. Reflux fills and mildly dilates the ureter and the collecting system.
5. Massive reflux grossly dilates the collecting system. All the calices are blunted with a loss of papillary
impression, and intrarenal reflux may be present. There is significant ureteral dilation and tortuosity.

158. What is vesicoureteral reflux grade 5?


1. Reflux only fills the ureter without dilation.
2. Reflux fills the ureter and the collecting system without dilation.
3. Reflux fills and mildly dilates the ureter and the collecting system.
4. Massive reflux grossly dilates the collecting system. All the calices are blunted with a loss of papillary
impression, and intrarenal reflux may be present. There is significant ureteral dilation and tortuosity.
5. Reflux fills and grossly dilates the ureter and the collecting system with blunting of the calices.
159. What are the risk factors for pediatric acute kidney injury?
1. Critically ill patients, nephrotoxin use, comorbid conditions
2. All children < 5 years, fever, comorbid conditions
3. Critically ill adolescents, fever, comorbid conditions
4. Critically ill patients > 5 years, hypertension, fever
5. All children < 5 years, comorbid conditions, hypertension

160. What is pollakiuria (daytime frequency) in a patient with voiding/ bladder dysfunction?
1. Abnormally frequent small voids in a previously toilet-trained child with no evidence of polyuria or urinary
tract infection
2. Uncontrolled leakage of urine, which can be continuous or intermittent
3. The sudden and unexpected experience of an immediate need to void
4. Difficulty in the initiation of voiding
5. Observed behavior used to either postpone voiding or suppress urgency, including standing on tiptoe and
forcefully crossing the legs

161. What is incontinence in a patient with voiding/ bladder dysfunction?


1. Abnormally frequent small voids in a previously toilet-trained child with no evidence of polyuria or urinary
tract infection
2. The sudden and unexpected experience of an immediate need to void
3. Uncontrolled leakage of urine, which can be continuous or intermittent
4. Difficulty in the initiation of voiding
5. Observed behavior used to either postpone voiding or suppress urgency, including standing on tiptoe and
forcefully crossing the legs

162. What is urgency in a patient with voiding/ bladder dysfunction?


1. The sudden and unexpected experience of an immediate need to void
2. Uncontrolled leakage of urine, which can be continuous or intermittent
3. Abnormally frequent small voids in a previously toilet-trained child with no evidence of polyuria or urinary
tract infection
4. Difficulty in the initiation of voiding
5. Observed behavior used to either postpone voiding or suppress urgency, including standing on tiptoe and
forcefully crossing the legs

163. What is hesitancy in a patient with voiding/ bladder dysfunction?


1. Abnormally frequent small voids in a previously toilet-trained child with no evidence of polyuria or urinary
tract infection
2. The sudden and unexpected experience of an immediate need to void
3. Uncontrolled leakage of urine, which can be continuous or intermittent
4. Difficulty in the initiation of voiding
5. Observed behavior used to either postpone voiding or suppress urgency, including standing on tiptoe and
forcefully crossing the legs

164. What is holding maneuver in a patient with voiding/ bladder dysfunction?


1. Difficulty in the initiation of voiding
2. The sudden and unexpected experience of an immediate need to void
3. Uncontrolled leakage of urine, which can be continuous or intermittent
4. Observed behavior used to either postpone voiding or suppress urgency, including standing on tiptoe and
forcefully crossing the legs
5. Abnormally frequent small voids in a previously toilet-trained child with no evidence of polyuria or urinary
tract infection

165. What is volume resuscitation in hypovolemic patients with prerenal acute kidney injury?
1. Furosemide (2-4 mg/kg)
2. Intravenous administration of isotonic saline, 20 mL/kg over 30 min
3. Sodium polystyrene sulfonate resin (Kayexalate), 1 g/kg
4. Calcium gluconate 10% solution, 100 mg/kg/dose
5. Administration of hypertonic (3%) saline

166. What is the treatment for severe hyperkalemia (serum potassium >7 mEq/L) in patients with acute kidney
injury?
1. ACE inhibitors/angiotensin receptor blockers (ARBs)
2. Calcium gluconate 10% solution, 100 mg/kg/dose
3. Intravenous administration of isotonic saline, 20 mL/kg over 30 min
4. Administration of hypertonic (3%) saline
5. Isradipine (0.05-0.15 mg/kg/dose)

167. What is the treatment for anemia in patients with chronic kidney disease?
1. Erythropoiesis-stimulating agents 4. Phosphate binders
2. ACE inhibitors/angiotensin receptor blockers 5. Active vitamin D sterol agents (Calcitriol)
3. Recombinant human growth hormone

168. What is the treatment for short stature in children with CKD who remain less than −2 SD for height despite
optimal medical support (adequate caloric intake and effective treatment of renal osteodystrophy, anemia, and
metabolic acidosis)?
1. Erythropoiesis-stimulating agents 4. Phosphate binders
2. ACE inhibitors/angiotensin receptor blockers 5. Active vitamin D sterol agents (Calcitriol)
3. Recombinant human growth hormone
169. Which of the following conditions is most likely to present with acute kidney injury?
1. Pyelonephritis 4. Diabetic nephropathy
2. Vesicoureteral reflux 5. Cystitis
3. Lupus nephritis

170. A child of 2 months was delivered to the clinic on the 5th day from the onset of the disease. He became acutely
ill: the temperature rose to 38.2 C, an unproductive cough appeared, the child began to refuse food. Upon
admission to the clinic, the temperature is 38.5 C, BR = 70 per minute, shortness of breath with a predominance of
the expiratory component. The skin is pale, perioral cyanosis is pronounced. There is a boxy sound above the lungs,
the breathing is hard, on both sides there is an abundance of small-bubbly wheezing on inhalation and exhalation.
No focal changes were detected on the X-ray of the chest organs. Your diagnosis:
1. Aspiration pneumonia 4. Foreign body
2. Bronchitis 5. Bronchiolitis
3. Bronchial asthma

171. Cabbage, oatmeal and buckwheat porridge, cheese, butter, baked potatoes, raisins, prunes and pumpkin were
included in the diet of a 9-year-old child. In what disease is such a diet justified?
1. Peptic ulcer disease 4. Rheumocarditis
2. Acute pneumonia 5. Bronchial asthma
3. Dysmetabolic nephropathy with oxalaturia

172. A 9-year-old child has been ill for three years. Complaints of abdominal pain, more in the right hypochondrium,
subfebrility. A positive symptom of Kera. In blood tests, an increase in ESR is noted. What disease causes such a
clinical picture:
1. Chronic enterocolitis 4. Biliary dyskinesia
2. Chronic cholecystitis 5. Acute intestinal infection
3. Chronic duodenitis

173. A 2-month-old baby with central cyanosis presented with the following findings on chest X-ray and ECG.
What is the most probable underlying diagnosis?
1. Tetralogy of Fallot 4. Ebstein’s anomal
2. Hypertrophic left heart syndrome 5. Hypertrophic right heart syndrome
3. Coarctation of aorta

174. A 7-year-old boy developed small hypopigmented depressed scars after fingernail scratches; he has been treated
with nonsteroidal antiinflammatory drugs (NSAIDs) for a rheumatic disease 6 weeks ago. Of the following, the
NSAID that is MOST likely to cause such a unique skin reaction is
A. Celecoxib D. Naproxen
B. Meloxicam E. Ibuprofen
C. Indomethacin

175. The gallbladder is congenitally absent in approximately 0.1% of the population. Hypoplasia or absence of the
gallbladder can be associated with
1. Biliary dyskinesia 4. Cholestasis
2. Cystic fibrosis 5. Wilson disease
3. Cirrhosis
176. The recurrent aspiration of small quantities of gastric, nasal, or oral contents can lead to several clinical
presentations, including recurrent bronchitis or bronchiolitis; recurrent pneumonia; atelectasis; wheezing; cough;
apnea; and/or laryngospasm. The MOST common underlying problem associated with recurrent pneumonias in
hospitalized children is
1. Oropharyngeal incoordination 4. Poor oral hygiene
2. Esophageal foreign body 5. Bronchopulmonary dysplasia
3. Nasoenteric tube

177. Recurrent pneumonia is defined as:


A. 2 or more episodes in a single year, without radiographic clearing between occurrences
B. 2 or more episodes in a single year, with radiographic clearing between occurrences
C. 3 or more episodes in a single year, with radiographic clearing between occurrences
D. 3 or more episodes in a single year, without radiographic clearing between occurrences
E. 4 or more episodes in a single year

178. Hydroxychloroquine sulfate is an antimalarial drug important in the treatment of SLE and dermatomyositis,
particularly cutaneous manifestations of disease and to reduce lupus flares. Of the following, the MOST important
procedure that should be done routinely during the course of administration is
1. Gastric endoscopy 4. Glucose-6-phosphate dehydrogenase enzyme
2. Bone marrow examination level assay
3. Muscle biopsy 5. Ophthalmological examination

179. You are meeting parents of a 14-year-old girl who has been treated with a monthly intravenous
cyclophosphamide for SLE-associated renal failure for the last 6 months; the mother is asking about the long-term
complications of this drug. All the following are long-term complications EXCEPT
A. Bone marrow suppression D. Lymphoma
B. Bladder cancer E. Infertility
C. Leukemia

180. A 4-year-old girl recently diagnosed with persistent oligoarticular juvenile idiopathic rheumatoid arthritis (JIA);
she has 3 involved joints including the right knee, right ankle, and left elbow; antinuclear antigen (ANA) is
significantly positive. Of the following, the MOST important step in the management of this girl is
A. Regular examination of locomotor system D. Periodic erythrocyte sedimentation rate (ESR)
B. Periodic slit-lamp examination monitoring
C. Periodic ANA monitoring E. Frequent С-reactive protein (CRP) monitoring

181. Antinuclear antigen (ANA) measurement test is useful in some rheumatologic diseases especially with persistent
oligoarticular juvenile idiopathic rheumatoid arthritis (JIA). All the following are more likely to be correlated with
ANA positivity EXCEPT
a. Anterior uveitis d. Symmetrical arthritis
b. Younger age at disease onset e. Lower number of involved joints over time
c. Female sex

182.Cyanosis in the newborn may be caused by which of the following:


1) Transposition of the great arteries. 4) Coarctation of the aorta.
2) VSD 5) Eisenmenger syndrome
3) Hyperbilirubinaemia .

183. The child with polyarticular JIA often has a more prolonged course of active joint inflammation and requires
early and aggressive therapy. Of the following, the predictor that carries the WORST prognosis is:
A. Old age at onset D. Small numbers of affected joints
B. Rheumatoid factor (RF) seronegativity E. Hip joint involvement
C. Absence of rheumatoid nodules

184. Drug-induced lupus refers to the presence of SLE manifestations triggered by exposure to specific medications,
including antibiotics. Of the following, the drug that is MOST likely associated with drug-induced lupus is
A. Isoniazid D. Penicillin
B. Rifampin E. Tetracycline
C. Nitrofurantoin

185. Systemic lupus erythematosus (SLE) is often characterized by periods of flare and disease quiescence or may
follow a more smoldering disease course. All the following lab tests correlate with active disease EXCEPT
a) high erythrocyte sedimentation rate d) positive anti-nuclear antibody titer
b) positive anti–double-stranded DNA level e) elevated C-reactive protein (CRP) value
c) low serum complement level

186. Rheumatoid factor (RF)–positive polyarthritis is characterized by aggressive symmetric inflammation of joints
of both upper and lower extremities. Of the following, the extra articular manifestation that is almost exclusively occur
in RF positive individuals is
A. Fever D. Extensor surfaces nodules
B. Evanescent rash E. Pericarditis
C. Uveitis

187. Chronic kidney disease in children <5-yr-old is MOST commonly a result of:
A. Lupus nephritis
B. Familial juvenile nephronophthisis
C. Alport syndrome
D. Focal segmental glomerulosclerosis
E. Autosomal dominant polycystic kidney disease

188. Despite the use of antibiotic agents, mortality remains high, in the range of 20-25%. Serious morbidity occurs
in 50-60% of children with documented infective endocarditis. Of the following, the MOST common morbidity is
A. Heart failure D. Acquired ventricular septal defect
B. Pulmonary emboli E. Heart block
C. Mycotic aneurysms

189. The definition of hypertension in children is


1) Average systolic blood pressure (SBP) and/or diastolic BP that is ≥95 th percentile for age, sex, on ≥3 occasions
2) Average systolic blood pressure (SBP) and/or diastolic BP that is ≥95th percentile for age, sex, and height on ≥2
occasions
3) Average systolic blood pressure (SBP) and/or diastolic BP that is ≥95 th percentile for age, sex, and height on ≥3
occasions
4) Average systolic blood pressure (SBP) and/or diastolic BP that is ≥90th percentile for age, sex, and height on ≥3
occasions
5) Average systolic blood pressure (SBP) and/or diastolic BP that is ≥99 th percentile for age, sex, and height on ≥3
occasions
190. What are some laboratory results you may find in a child who has acute poststreptococcal glomerulonephritis?
1. A urine sample positive for glucose and hematuria; a decreased blood urea nitrogen (BUN); and a normal
glomerular filtration rate (GFR)
2. A urine sample negative for proteinuria and hematuria; a decreased blood urea nitrogen (BUN); a decreased
creatinine level; and a normal glomerular filtration rate (GFR)
3. A urine sample positive for increased sodium, potassium, and glucose; and an increased glomerular filtration rate
(GFR)
4. A urine sample positive for proteinuria and hematuria; an increased creatinine level; and a decreased glomerular
filtration rate (GFR)
5. A urine sample positive for proteinuria and hematuria; a decreased blood urea nitrogen (BUN); a decreased
creatinine level; and a normal glomerular filtration rate (GFR)

191. What is the treatment plan for a 6-year-old boy who has acute poststreptococcal glomerulonephritis?
1. Supportive treatment based on the symptoms
2. Treatment with corticosteroid medications
3. Treatment with anti-inflammatory medications
4. Treatment with cytotoxic medications
5. Treatment with biological agents

192. What are the signs and symptoms of acute poststreptococcal glomerulonephritis?
1. Edema, hypertension, polyuria, hematuria and proteinuria
2. Edema, hypotension, oliguria, hematuria and glucosuria
3. Edema, hypertension, crystalluria, hematuria and proteinuria
4. Edema, hypertension, oliguria, hematuria and proteinuria
5. Edema, hypotension, oliguria, pyuria and proteinuria

193. Which of the following conditions most likely causes hematuria?


1. Porphyria
2. Tyrosinemia
3. Rhabdomyolysis
4. Post-streptococcal glomerulonephritis
5. Hyperbilirubinemia

194. Which of the following is associated with post-streptococcal glomerulonephritis?


1. Elevated C3, C4, and CH50 complement levels
2. Low blood pressure
3. Protein-to-creatinine ratio of <0.2
4. Hypotension
5. Elevated antistreptolysin O (ASO) titers

195. Which client is most at risk of developing acute streptococcal glomerulonephritis?


1. A 10-year-old child with the mumps
2. A 6-year-old child with impetigo
3. A 15-year-old girl with influenza
4. An 8-year-old boy with the chickenpox
5. A 17-year-old girl with the systemic lupus erythematosus

196. A 6-year-old boy was diagnosed with a streptococcal infection in the throat. The child was sent home and began
taking antibiotics. During health teaching, the boy's mother was told to monitor the child's urine output because the
boy was at risk of developing acute streptococcal glomerulonephritis. What is the time frame for developing this
complication of poststreptococcal throat infection?
1. 1–2 weeks
2. 2–3 weeks
3. 3–4 weeks
4. 4–5 weeks
5. 6-7 weeks

197. Which statement about poststreptococcal glomerulonephritis is correct?


1. It results from a viral infection of the throat or skin and tends to present 6 weeks after the infection.
2. Clients are less likely to experience hematuria with this condition.
3. Antibodies develop and deposit within the kidney, and they can cause inflammation.
4. Clients with this condition should consume a diet rich in sodium to prevent hyponatremia.
5. The body develops antibodies, which deposit in the ureters, causing urine to back up into the kidney and result in
inflammation.

198. How does a streptococcal infection interfere with the functioning of the kidneys?
1. The body develops antigens, which deposit in the nephron.
2. The body develops antibodies, which deposit in the renal veins and cause inflammation.
3. The body develops antibodies, which deposit in the ureters, causing urine to back up into the kidney and result in
inflammation.
4. The body develops antibodies, which deposit in the glomerulus and cause inflammation.
5. The body develops antibodies, which deposit in the collecting ducts and cause inflammation.

199. Which of the following is an indication for imaging in urinary tract infection?
1. Failure to improve with antibiotic therapy
2. Acute uncomplicated urinary tract infection
3. Acute pyelonephritis
4. Acute cystitis
5. Clinically relevant uropathogens in urine culture

200. Which substances tested for in the urine dipstick test indicate a urinary tract infection?
1. Leukocytes, ketones
2. Protein, nitrites
3. Leukocytes, glucose
4. Leukocytes, nitrites
5. Bilirubin, nitrites

201. Why are girls more prone to urinary tract infections than boys?
1. Girls have longer urethras and larger bladders
2. Girls tend to urinate less frequently
3. Girls are likely to have more bacteria due to the close proximity to the vagina
4. The propensity of bacterial attachment to the female periurethral mucosa and shorter urethras may account for
this difference
5. Only in girls the bacteria causing cystitis ascend to the kidney to cause pyelonephritis

202. Which of the following best defines pyelonephritis?


1. Infection of the entire urinary tract
2. Cystitis complicated by fever
3. Perirenal abscess formation
4. Infection of the kidneys/upper urinary tract
5. Urinary tract infection with obstruction

203. Which of the following describes the correct pathway of infection during the pathogenesis of pyelonephritis?
1. Ureter, urethra, bladder, renal calyx, renal papilla
2. Urethra, bladder, ureter, renal papilla, renal calyx
3. Ureter, bladder, renal calyx, ureter, renal papilla
4. Urethra, bladder, ureter, renal calyx, renal papilla
5. Urethra, ureter, bladder, renal papilla, renal calyx

204. Which of the following is possible complication of vesicoureteral reflux?


1. Renal scarring
2. Renal stones
3. Cystitis
4. Renal vein thrombosis
5. Hypertension

205. Which of the following is required for a diagnosis of chronic kidney disease (CKD)?
1. Hematuria with dysmorphic red blood cells
2. Heavy proteinuria (protein excretion greater than 3.5 g/24 hours)
3. Oliguria (urine production less than 0.5 mL/m2/h)
4. Multiple cysts on renal imaging
5. Persistent evidence of kidney damage or a decrease in function for at least 3 months

206. Which of the following best describes the relationship between acute kidney injury (AKI) and chronic kidney
disease (CKD)?
1. Pre-existing CKD decreases the risk of AKI.
2. AKI and CKD are two separate, mutually exclusive entities.
3. AKI can slow the progression of pre-existing CKD.
4. Unresolved AKI may lead to CKD.
5. AKI decreases the risk of CKD.

207. Which hormone is a part of a hormone system that regulates blood pressure, fluid and electrolyte balance, and
systemic vascular resistance?
1. Angiotensin-II
2. Atrial natriuretic peptide
3. Parathyroid hormone (PTH)
4. 25OH Vitamin D
5. 3-Dehydroretinol

208. Which of the following is an indication for kidney replacement therapy in patients with CKD?
1. GFR = 75 mL/min/1.73 m^2
2. Subjective findings of uremia (fatigue, weakness, nausea, vomiting, anorexia, and poor sleep patterns)
3. Age > 17 years
4. Serum creatinine < 1.2 mg/dL
5. Controlled electrolyte abnormalities

209. Which of the following is an indication for kidney replacement therapy in patients with CKD?
1. GFR = 60 mL/min/1.73 m^2
2. Age < 1 year
3. Severe fluid restrictions that inhibit the ability to provide appropriate nutrition sufficient for linear growth
4. Serum creatinine = 1.5 mg/dL
5. Controlled electrolyte abnormalities

210. Which of the following is true regarding the staging of CKD?


1. A low glomerular filtration rate measurement is required to diagnose CKD.
2. A patient diagnosed with stage 3 CKD has very little chance of disease progression.
3. Patients with stage 1 CKD should be treated with kidney replacement therapy.
4. It is intended to identify those at greatest risk for progression and complications.
5. Earlier stages of CKD are usually symptomatic.

211. What is the cause of anemia seen in patients with CKD?


1. Decreased red blood cell production
2. Excessive sequestration of red blood cells in the spleen
3. Vitamin deficiency
4. Deficient globin synthesis
5. Red cell membranopathy

212. Which of the following is involved in the pathogenesis of mineral and bone disorders seen in CKD patients?
1. Increased glomerular filtration rate (GFR)
2. Inadequate activation (hydroxylation) of vitamin D
3. Decreased parathyroid hormone (PTH) activity
4. Increased calcitriol activity
5. Decreased phosphatonin activity

213. Which type of kidney replacement therapy has the most favorable outcome in CKD patients?
1. Kidney transplantation
2. Home hemodialysis
3. Peritoneal dialysis
4. In-center hemodialysis
5. Continuous hemodiafiltration

214. Which of the following is a limitation of measuring protein excretion by using a 24-hour urine collection?
1. It is difficult to perform on a regular basis.
2. It has low sensitivity to albumin.
3. It can be influenced by variations in protein production throughout the day.
4. It might overestimate proteinuria in women.
5. It can be performed only in the inpatient setting.

215. Which of the following explains why the serum creatinine level is a useful measure for estimating GFR?
1. Creatinine is freely filtered across the glomerulus and is neither reabsorbed nor metabolized by the kidneys.
2. Serum creatinine can be used to estimate the GFR in individuals with unstable kidney function.
3. Creatinine production remains constant among individuals with significant variations in muscle mass.
4. Serum creatinine is less affected by sex, age, and muscle mass than cystatin C.
5. Serum creatinine is not secreted by the renal tubules.

216. A patient presents with fever, eosinophilia, elevated creatinine, and proteinuria after starting penicillin for a
Streptococcus infection and Ibuprofen because of fever. Which type of acute kidney injury is most likely?
1. Prerenal 4. Renovascular disease
2. Intrinsic renal 5. Due to increased renal perfusion
3. Postrenal

217. A child presents with a 2-day history of vomiting and diarrhea. He has oliguria and elevated creatinine. Which
type of acute kidney injury (AKI) is most likely?
1. Prerenal 3. Postrenal
2. AKI related to acute postinfectious 4. Renovascular disease
glomerulonephritis 5. AKI due to increased renal perfusion

218. A 6 yr old child with a recent pharyngitis presents with periorbital edema, hypertension, gross hematuria,
oliguria, and elevated creatinine. Which type of acute kidney injury (AKI) is most likely?
1. Prerenal AKI due to decreased renal perfusion
2. Postrenal AKI related to anatomic obstructions to the lower urinary tract
3. Intrinsic renal AKI related to acute postinfectious glomerulonephritis
4. Renovascular disease
5. Acute tubular necrosis

219. A neonate with a history of hydronephrosis seen on prenatal ultrasound studies presents with a palpable bladder
and elevated creatinine. Which type of acute kidney injury (AKI) is most likely?
1. Postrenal AKI related to congenital urinary tract obstruction, probably posterior urethral valves
2. Intrinsic renal AKI related to acute postinfectious glomerulonephritis
3. Prerenal AKI due to decreased renal perfusion
4. Renovascular disease
5. Acute tubular necrosis

220. When a patient presents with true volume depletion and prerenal acute kidney injury, which treatment is most
appropriate?
1. Diuretics 4. Intravenous fluids
2. Albumin 5. Salt and water restriction
3. Vasopressor support

221. Which of the following is the most common cause of acute tubular necrosis?
1. Inadequate electrolyte intake 4. Decrease in angiotensin levels
2. Decreased renal perfusion 5. Infections
3. Urinary retention causing obstruction

222. Which of the following conditions is most likely to present with acute kidney injury?
1) Pyelonephritis 4) Diabetic nephropathy
2) Vesicoureteral reflux 5) Cystitis
3) Lupus nephritis

223. "A 3-month-old child, against the background of subfebrile body temperature and rhinitis, has pallor, cyanosis
of the nasolabial triangle, severe expiratory dyspnea, deflated chest, dry cough, participation of auxiliary muscles in
breathing. and small bubbling wet rales on both sides. In the blood: Hb - 112 g / l
1) Acute bronchiolitis 4) Bronchial asthma, exacerbation
2) Acute (simple) bronchitis 5) Acute obstructive bronchitis
3) Bilateral pneumonia

224. A 12-year-old female adolescent has a recent diagnosis of SLE; she has a faint malar rash and mild arthritis
including both elbow and knee joints as well as the joints of small fingers. Of the following, the MOST appropriate
initial therapy for this patient is
A. Systemic steroids D. Cyclophosphamide
B. Methotrexate E. Hydroxychloroquine
C. Leflunomide

225. The initial period of rickets is mostly diagnosed by:


A. 1 month of life D. 5-7 months of life
B. 2-3 months of life E. after a year
C. 3-5 months of life

226. With rickets, deformation of the lower extremities occurs in:


A. 3 – 6 months D. 1 – 3 months
B. 9 – 12 months E. 0 – 1 month
C. 6 - 9 months

227. Common to both acute and chronic malnutrition is:

a) Weight for age c) Height for age


b) Weight for height d) Height for weight
e) Height for length

228.What gastroenterological diseases prevalent in children?


a) Isolated gastritis d) Duodenal ulcer
b) An isolated duodenitis e) Gastro duodenitis
c) Peptic ulcer.

229.The boy, 8 years old, has pale skin, general weakness, and decreased appetite. In general blood test: Hb - 80 g/l,
RBC 3,86x10 9/l , CI 0,7, WBC - 6x10 9/l, bands - 2%, eosynophiles – 5%, segments - 51%, lymph. - 32%, monocytes
-10%, thromb. - 210х10 9/l ESR - 7 mm/hour. What disease is possible in tis case?
a) Iron-deficiency anemia d) B12-deficiency anemia
b) Hemolytic anemia e) Aplastic anemia
c) Acute leukemia
230. Have 4-month girl child complains of anxiety, sleep fitfully, poor appetite. The baby was born premature from a
second pregnancy weight at the time of birth was 2400 g, the skin and visible mucous membranes pale. Heart rate -
144 in 1 min. Heart sounds loud, rhythmic activity, tachycardia, mild systolic murmur. The liver serves 4 cm from the
edge of costal arch. Analysis of blood: red blood cells - 2.5 * 10 12 degree / l, hemoglobin - 88 g / l, color index -
0.72, reticulocytes - 0.7%. What kind of anemia should think?
a) Iron deficiency anemia severity, Hypochromic
b) Iron deficiency anemia severity, hyperchromic.
c) Hypoplastic anemia.
d) Iron deficiency anemia and severity
e) Aplastic anemia

231. The patient, 15 years old, 5 years suffering from duodenal ulcer. After exercise felt severe pain in the epigastric
region and right upper quadrant. It was vomiting. There is a weak pulse, sharp pain from the navel to the right, the
disappearance of hepatic dullness during percussion. What caused condition that requires emergency care?
a) Duodenal ulcer perforation d) Exacerbations of cholelithiasis
b) Spontaneous pneumothorax e) Pyloroduodenal stenosis
c) Aggravation of kidney stones (renal colic)

232. Hypotrophy (CND) degree III (Severe malnutrition) characterized by:


a) Deficit weight of standards for10 – 20% d) Deficit weight of standards 30 – 60%
b) Deficit weight of standards for 20 – 30% e) Deficit weight of standards 30 – 50%
c) Deficit weight of standards more than 30%

233. The child, 2 months, was born in October, weighing 3000 g, is on the breastfed. Now the weight is 4400 grams,
neuro-psychological development age. During the inspection, clean skin, pink; organs and systems - no change. What
is the prevention of rickets be assigned to the child?
a) Vitamin D 500 IU per day. d) Calcium gluconate 400-500 mg per day.
b) Vitamin D to 2000 IU per day. e) Vitamin D 1000 IU per day.
c) Ascorbic acid 30 mg 3 times a day.

234. Define the characteristic feature of subacute rickets.


a) Kraniotabes d) The presence of severe hypocalcemia
b) Prevalence of osteomalacia e) Rickety rosary
c) Prevalence of osteoid hyperplasia

235. Define the calcium content in serum at hypervitaminosis D.


a) Calcium levels above 2.7 mmol / l d) Calcium levels upper than 1.8 mmol / l
b) Within the physiological oscillations. e) Calcium levels less than 1.8 mmol / l
c) Calcium levels lower than 2.2 mmol / l

236. The first level of diet therapy at hypotrophy is called:


a) Rest and minimum diet d) Primary diet
b) Increase in the caloric input (intermediary) e) Secondary diet
c) Returning to normal regime

237. The earliest sign of the development of vitamin d-deficient rickets is


a) Deformity of the lower extremities d) Vegetative disorders
b) Delayed physical development e) Mental retardation
c) Deformation of the skull bones

238. A pediatrician examines a healthy full-term baby at the age of 1 month, who is on natural feeding. What kind of
disease prevention will the doctor recommend first?
a) Iron deficiency anemia d) Rickets
b) Hypervitaminosis D e) Dystrophy
c) Acute respiratory viral infection

239. The child was hospitalized for 11 months due to ricketogenic tetany. After the emergency treatment, the condition
stabilized, the seizures did not recur. Determine further therapeutic tactics.
1. Prescribe large doses of vitamin D 4. Prescribe anticonvulsant therapy
2. Prescribe large doses of calcium 5. Prescribe calcium supplements and anti- rickets
3. Prescribe glucocorticoids treatment

240. The child is 1.5 months old. He was born prematurely with a weight of 2000g. During pregnancy, the mother
suffered from gestosis of the 1st and 2nd half of pregnancy. For 2 weeks, there is increased sweating, excitability.
What is the most likely pathological condition? The consequences of perinatal damage to the nervous system.
1. Rickets. 4. The initial manifestations of acute respiratory
2. Functional disorder of the gastrointestinal tract. viral infection
3. Spasmophilia. 5. Iron deficiency anemia

241. The pediatrician, after examining 2-month-old Masha, determined rickets, first degree of severity, initial
period, acute course. What is the daily dose of an aqueous solution of vitamin D3 needed for treatment?
1. 2000 IU (4 drops) 4. 5000 IU (10 drops)
2. 3000 IU (6 drops) 5. 6000 IU (12 drops)
3. 4000 IU (8 drops)

242. The mother of 3-month-old Sasha complains of restless sleep in the child, decreased appetite. On examination:
flattening of the occiput, softening of the edges of the large fontanel, hypocalcaemia in the blood test, Hb – 110 g / l,
ESR – 8 mm / hour. Which conclusion is correct?
1. Anemia. 4. Phosphate is diabetes
2. Epilepsy 5. Rickets, acute period
3. Spasmophilia. 6. Iron deficiency anemia

243. In a child of 5 months. against the background of rickets treatment (oral vitamin D3 in a dose of 5 thousand IU
and total Ultraviolet irradiation) there was an attack of tonic-clonic seizures. Which examination should be
prescribed first?
1. Determination of the level of calcium and phosphorus in the blood.
2. Determination of the level of calcium and phosphorus in the urine
3. Spinal tap
4. Neurosonography
5. Computed tomography of the brain

244. A 3-month-old child was diagnosed with rickets. Which acid plays an important role in the metabolism of
phosphorus and calcium, and affects the action of calciferol, the function of the parathyroid glands?
1. Acetic acid 4. Sulfuric acid
2. Salt 5. Lemon
3. Phosphoric

245. The parents of a 6-month-old child are concerned about the rapid growth of his head, since the hats bought earlier
turned out to be small. The measurements carried out by the nurse showed that the circumference of the child's head
is 43 cm, the chest is 45 cm. Evaluate these parameters.
1. Proportional and appropriate to the age of the 3. Increased head size
child 4. The chest is enlarged
2. Both parameters have been increased 5. Both parameters are below the age limit

246. A child with manifestations of rickets deficiency developed laryngospasm and carpopedal spasm after
prolonged exposure to the spring sun. This is typical for:
1. Febrile seizures 4. Hypoparothyroiditis
2. Encephalitis 5. Ricketogenic tetany
3. Meningitis

247. Hereditary rickets-like diseases include


1. Vitamin D-dependent rickets, vitamin D-resistant rickets, de Toni-Debre-Fanconi disease, galactosemia
2. Vitamin D-dependent rickets, vitamin D-resistant rickets, de Toni-Debre-Fanconi disease, renal tubular acidosis
3. Vitamin D-dependent rickets, vitamin D-resistant rickets, cystic fibrosis, renal tubular acidosis
4. Nonspecific ulcerative colitis, vitamin D-resistant rickets, de Toni-Debre-Fanconi disease, renal tubular acidosis
5. Vitamin D-dependent rickets, renal tubular acidosis, vitamin D-resistant rickets, de Toni-Debre-Fanconi disease

248. A pediatrician examined a two-month-old child. Mother's complaints of periodic anxiety, excessive sweating.
On examination: the back of the head is flattened, bald. The edges of the large fontanelle are pliable. What kind of
disease are we talking about?
1. Vitamin D-resistant rickets
2. Phosphate diabetes
3. Debre-de Toni-Fanconi syndrome
4. Rickets
5. Spasmophilia

249 A month-old child has anxiety, increased sweating of the head. From the anamnesis of life: since his birthday
(September 5), he has been fed cow's milk. Upon examination, craniotabes is noted. The doctor prescribed a course
of UV radiation. Decide on the need to prescribe vitamin D3 preparations to this child.
1. 2 – 2.5 months after the end of the course of Ultraviolet irradiation
2. There is no need
3. Together with the Ultraviolet irradiation
4. Immediately after completing the course of the Ufa
5. 1 month after the end of the Ultraviolet irradiation course

250. A 3-month-old child has baldness of the back of the head, restless sleep, increased sweating. What kind of
pathological condition can you think of?
1. Anemia 4. Phosphate is diabetes
2. Spasmophilia 5. Chondrodystrophy
3. Rickets

251. The boy is 9 months old. during the scream, noisy breathing appeared, cyanosis of the skin, cold sweat, short-
term apnea, tonic cramps in the arms and legs. After a few minutes, the child became active again. Upon examination,
only signs of rickets were revealed, body temperature – 36.6 ° C. It is fed with breast milk. Which drug should be
prescribed first after an attack.
1. Vitamin C 4. Sodium Oxybutyrate
2. Vitamin D 5. Finlepsinum
3. Calcium Gluconate

252. In a 3-year-old child, when applying to a preschool institution, changes in the bone system were revealed:
thickenings on the ribs at the junction of the bone and cartilaginous parts - "rosaries", deformation of the sternum, X-
shaped deformities of the lower extremities. The child was staying with his grandmother until he was 3 years old, and
was not observed by the family doctor. Make a plan for follow-up and rehabilitation measures.
1. Prednisone with a short course of 5-7 days, 1-2 mg / kg of body weight per day, exclude foods rich in calcium,
vitamin E, C, group B. Prevention: strict observance of the rules, instructions for the use of the dosage form of
vitamin D.
2. Control of physical and neuropsychiatric development, in case of deviation - exercise therapy, massage, calcium
D3 Nicomed
3. Introduction of calcium–enriched foods into the diet
4. Immediate hospitalization, after clarification of the diagnosis – dispensary registration, monitoring of
development and implementation of recommendations of specialists
5. Control of physical and neuropsychiatric development, in case of deviation - exercise therapy, massage, calcium
D3 Nicomed, daily stay in the fresh air, the introduction of calcium–enriched foods into the diet

253. Given the following data, determine the clinical course of rickets characterized by hypertrophy of the frontal and
parietal eminences, thickening of the interphalangeal joints of the fingers: skeletal deformities, growth retardation,
bone pain and tenderness, dental anomalies, muscle weakness, muscle cramps, seizures:
1. Nutritional Rickets 4. Rickets of Prematurity
2. Hypophosphatemic Rickets 5. Vitamin D Resistant Rickets
3. Calcipenic Rickets

254. A 2-year-old boy with severe impairments in physical and psychomotor development and skeletal deformities is
suspected to have Debre-de Toni-Fanconi syndrome. What changes in urine confirm this diagnosis?
1. Galactosuria, lactosuria 4. Proteinuria, hematuria
2. Glucosuria, hyperaminoaciduria 5. Cylindruria, bacteriuria
3. Acetonuria, isostenuria

255. The child was 6 months old. against the background of a mild course of Acute respiratory infection, repeated
clonic seizures appeared. Upon examination, signs of rickets of moderate severity were diagnosed. Blood calcium is
1.2 mmol/l. There is no data on perinatal damage to the central nervous system. The cerebrospinal fluid parameters
are normal. Artificial feeding. What is the most likely condition?
1. Neurotoxicosis 4. Encephalitic reaction
2. Meningitis 5. Spasmophilia
3. Encephalitis

256. 1.8-year-old boy was brought to the doctor by his mother complaining of frequent abdominal pain and diarrhea
after consuming dairy products. Examination revealed reduced levels of lactase in the intestines. Which of the
following conditions is most likely in the child?
1. Selective IgA deficiency 4. Undifferentiated immunoglobulin enteropathy
2. Cow's milk allergy 5. Lactase deficiency
3. Polycystic ovary syndrome

257.A 5-year-old girl named Anya was brought to the emergency department by her mother with complaints of chronic
abdominal pain, diarrhea, and poor growth. Upon examination, she was found to exhibit fatigue and irritability, as
well as nutrient deficiencies including iron and vitamin D. Serological tests revealed elevated levels of anti-tissue
transglutaminase antibodies (anti-tTG) and anti-endomysial antibodies. A small bowel biopsy confirmed the presence
of villous atrophy. Which of the following conditions is most likely in the child?
1. Selective IgA deficiency 4. Undifferentiated immunoglobulin enteropathy
2. Cow's milk allergy 5. Celiac disease
3. C.Polycystic ovary syndrome

258.Causes of prenatal hypotrophy:


1. Ectopic pregnancy 4. Chromosomal abnormalities
2. Hypoplasia of the pancreas 5. Maternal malnutrition
3. Phenylalanine intolerance

259.Which diseases can lead to secondary lactose intolerance due to damage to the mucous membrane of the small
intestine?
1. Celiac disease 4. Gut-associated lymphoid tissue (GALT)
2. Crohn's disease syndrome
3. Rotavirus infection 5. Irritable Bowel Syndrome (IBS)

260. Children with сhronic malnutritionshould discontinue therapeutic feeding only when they meet the following
criteria:
1. Weight/height or weight/length will be at least ≥ –2 SD
2. Mid-upper arm circumference will be < 125 mm
3. Absence of edema for 2 days
4. Mid-upper arm circumference will be < 50 mm
5. Weight/height or weight/length will be at least ≥ –4 SD

261. The maximum diagnostic program for detecting hypotrophy in malabsorption syndrome includes:
1. Comprehensive blood tests
2. Stool pH determination
3. Evaluation of fecal analysis
4. Collection and assessment of the child's biological and genealogical history
5. Analysis of pancreatic enzyme levels in stool (lipase, amylase).

262. Which form of prenatal dystrophy can be considered inconsistent with the other listed forms?
1. Neurotic 4. Encephalopathic
2. Neuroendocrine 5. Neurovegetative
3. Neurodystrophic

263. Hypostature is -
1. Excess body weight in relation to height 4. Uniform lag in body weight and height from age
2. Uneven lag in body weight and height from age norms
norms 5. Uniform lag in body weight relative to height
3. Deficiency of body weight relative to height

264. 4-year-old Maxim was brought to the doctor by his mother with complaints of edema, frequent abdominal pain,
and diarrhea. Upon examination, signs of edema were detected, including abdominal and lower extremity edema. The
child also appears apathetic and fatigued. Objective examination revealed a low level of albumin in the blood and the
absence of facial edema. Laboratory analyses showed a low level of proteins and hypoalbuminemia. Based on the
symptoms and laboratory data, Maxim is suspected of having kwashiorkor, a form of protein-energy malnutrition.
Which of the following treatment directions is most suitable for 4-year-old Maxim suspected of kwashiorkor?
1. Initiate feeding through a gastrostomy tube to ensure an adequate intake of nutrients
2. Recommend liver transplantation to correct metabolic disorders
3. Conduct urgent gastroenterological examination to detect possible complications
4. Recommend gastroenterological rehabilitation and long-term treatment in stationary conditions to stabilize the
condition
5. Prescribe intensive pharmacological therapy for rapid elimination of edema and normalization of digestion

265. Infant is 2 months old, born at term with a weight of 3500 g. Breastfeeding is partial. Currently weighs 4900 g.
Evaluate the infant's body mass.
a) According to age d) 2nd degree hypotrophy
b) 150 g below normal e) 3rd degree hypotrophy
c) 1st degree hypotrophy
266. For the 4-year-old child: At birth, the child weighed 3500 g, with a body length of 50 cm, head circumference
of 35 cm, and chest circumference of 32 cm. The mother consulted a doctor because the child is not gaining weight
well. What weight should the 4-year-old child have?
a) Weight 20 kg. d) Weight 16.5 kg.
b) Weight 15 kg. e) Weight 18 kg.
c) Weight 16 kg.

266.For the 5-month-old child: The child is 5 months old and is exclusively breastfed. During a check-up, the doctor
diagnosed the child with 1st degree hypotrophy. What is the most desirable first complementary food for this child?
a) Vegetable puree
b) 5% semolina porridge
c) 10% semolina porridge
d) Buckwheat porridge
e) Oatmeal porridge

267.Child is 4 months old, with a birth weight of 3350 g. Two months ago, the child experienced an acute intestinal
infection, after which unstable stool and regurgitation appeared. At the time of examination, the body weight is 5000
g, with decreased tissue turgor, thinning of subcutaneous fat layer on the torso and limbs. Muscular hypotonia,
hepatomegaly are observed. Preliminary diagnosis: 2nd degree hypotrophy. What volume of feeding is necessary for
this child?
a) 1/4 of the age-appropriate amount d) 1/3 of the age-appropriate amount
b) According to age e) 1/2 of the age-appropriate amount
c) 1/5 of the age-appropriate amount

268.A 4-month-old child has been diagnosed with 2nd degree hypotrophy. What is the deficit in body weight for the
child?
a) 21-30% d) 5-10%
b) 10-20% e) 51-70%
c) 31-50%

269.Child is 30 days old, born prematurely with a body weight of 2300 g. The mother has hypogalactia, and the child
gained 300 g in the first month. What assumption would be optimal for the family doctor in this situation?
a) Supplementation with donor milk d) Transition to artificial feeding with adapted
b) Supplementation with 5% semolina porridge formulas
c) Transition to artificial feeding with cow's milk e) Observation over time

270.A 1.5-year-old child, born with a body weight of 3100 g and a body length of 51 cm, is breastfed. After introducing
complementary feeding (oatmeal porridge), the child stopped gaining weight, and there were feces with a strong
unpleasant odor in large quantities. Objectively, the child exhibits pale skin, an enlarged abdomen, and signs of 3rd
degree hypotrophy. What deficit in body weight corresponds to this degree of hypotrophy?
a) 5-10% d) 31% and above
b) 11-20% e) Doesn't matter
c) 21-30%

271.A 2-month-old girl is being transitioned to artificial feeding. She was born with a body weight of 3900 g.
Currently, her body weight is 3900 g. Specify the daily feeding volume for this child.
a) 650 ml d) 600 ml
b) 730 ml e) 800 ml
c) 750 ml

272.A full-term newborn boy was born with a body weight of 3800 g, and on the 3rd day of life, his body weight is
3200 g. Evaluate the weight loss in the child.
a) Doesn't matter d) Within the norm
b) Cannot be assessed e) Exceeds the norm
c) Below the norm

273.A 4-month-old child has been experiencing frequent watery stools from the first days of life. Breastfeeding is
natural. After consuming milk, the mother experiences abdominal pain and liquid stool. The child is active, but has a
24% deficit in body weight. Stool occurs 3-5 times a day, watery and acidic-smelling. Examination revealed: sweat
chloride 20.4 mEq/L, stool culture - negative. What is the probable cause of hypotrophy in the child?
a) Enteritis
b) Congenital lactose deficiency
c) Celiac disease
d) Cystic fibrosis
e) Exudative enteropathy

274.The child sleeps restlessly, gnashes his teeth in his sleep, combs the perianal area. Thin white lines with a length
of 1 cm and pointed ends were revealed. Find out the diagnosis?
a) Trichocephalosis d) Enterobiosis
b) Ascariasis e) Opisthorchiasis
c) Trichinosis

275. A 2-month-old girl is on artificial feeding. How often should the dynamics of body weight be monitored in a
satisfactory condition of the child to prevent hypotrophy?
a) Twice a week d) Once a month
b) Once every 2 weeks e) Twice a day
c) Once every 3 weeks

276.A family doctor reviews the basic anthropometric data of two girls, aged 13 and 15, from the same family. The
body mass index (BMI) is most commonly used for assessment. It is:
a) Body weight multiplied by height d) Height squared minus body weight
b) Body weight divided by height e) Body weight (kg) divided by height squared (m)
c) Height divided by body weight

277.A mother consulted a family doctor regarding her 2-week-old baby because he is gaining weight poorly. Prenatal
hypotrophy of the II degree was diagnosed in the full-term baby, characterized by the following weight-to-height ratio:
a) 80 d) 60-70
b) 75-80 e) 50-55
c) 70-75

278.A 6-month-old child with a birth weight of 3000 g, whose physical development corresponds to his age, is on
artificial feeding with adaptive formulas. Determine the daily food volume for the child.
a) 500 ml d) 900 ml
b) 600 ml e) 1000 ml
c) 700 ml

279.The child is 5 months old, with a birth weight of 3000 grams, currently weighing 5500 grams. They are partially
breastfed (receiving 500 ml of breast milk and 200 ml of "Baby" formula per day). The appetite is good, and there
are no digestive disorders. What is the cause of hypotrophy in this case?
a) Mother's hypogalactia d) Dysbacteriosis
b) Syndrome of impaired intestinal absorption e) Alimentary factor
c) Intestinal infection

280. In a patient with severe anemia and allergic phenomena, small mobile worms of red color with a size of 1 cm
were found in the feces. Tell the most appropriate diagnosis?
a) Ascariasis d) Loaosis
b) Hookworm e) Trichinosis
c) Dracunculosis

281.Helminth larvae were accidentally discovered in the laboratory tests during sputum microscopy
of a pneumonia patient. During the blood test, eosinophilia was detected. What kind of helminthosis can be
expected?
a) Vuhereriosis d) Paragonimiasis
b) Ascariasis e) Opisthorchiasis
c) Trichocephalosis

282.During puncture of a liver cyst (tumor with fluid), small nodular formations in the form of grains of sand were
revealed in a transparent, barely yellowish liquid. What kind of helminthiasis can be foreseen?
a) Echinococcosis d) Hymenolepiasis
b) Fascioliasis e) Cysticercosis
c) Schistosomiasis

283.Large oval yellowish eggs with a dark brown uneven shell were found in the feces of a patient with a digestive
disorder, a dark mass in the middle, and free spaces in the shape of a crescent at the poles. What is the diagnosis?
a) Teniasis d) Trichinosis
b) Ascariasis e) Trichocephaliasis
c) Fascioliasis

284.The patient has headache, muscle pain during movement, swallowing, chewing and eye rotation, weakness,
fever, swelling of the eyelids and face. There are no eggs in the feces and perianal area. What is the probable
helminthiasis?
a) Cysticercosis b) Trichinosis
c) Hookworm e) Trichocephalosis
d) Echinococcosis

285.Echinococcus was detected in laboratory rabbits during autopsy. A rabbit, like a human, is an additional host for
the parasite:
a) carrier d) reservoir host
b) intermediate host e) none of the above
c) final host

286.A sick child periodically has abdominal pain, loose stools, and nausea. One day, a cylindrical white worm of 15
cm was released with vomiting. What kind of laboratory test should be carried out?
a) Detection of segments in the feces
b) Detection of helminth larvae in the muscles by biopsy
c) Ovoscopy of the perianal zone by scraping or using adhesive tape
d) Examination of feces and duodenal contents on eggs
e) Fecal examination for eggs

287. White helminths of 5-10 mm were found in the feces, with a vesicular expansion of the esophagus in front of
them. The eggs were found not in feces, but in scraping from the perianal folds, colorless, non-symmetrical, oval.
What is the diagnosis?
a) Hookworm disease d) Trichinosis
b) Enterobiosis e) Trichocephalosis
c) Teniosis

288.For the prevention of which helminthiasis is it necessary to follow the rules of personal hygiene?
a) Fascioliasis d) Opisthorchiasis
b) Diphyllobothriosis e) Echinococcosis
c) Teniarinchiasis

289.A doctor put a preliminary diagnosis to an employee of a livestock farm: echinococcosis. The diagnosis was
confirmed during surgery. From which animal could the patient have contracted echinococcosis?
a) Pigs d) Dogs
b) Cows e) Cats
c) A rabbit

290.During the operation, small bubbles of small size with an insignificant amount of liquid were detected in the
patient's liver, which fit tightly one to the other. What kind of helminthiasis was revealed in the patient?
a) Fasciolosis d) Echinococcosis
b) Alveococcosis e) Dicroceliosis
c) Opisthorchiasis

291.Treatment of the patient with pneumonia did not significantly alleviate his condition. He began to complain of
pain in his stomach, nausea and other digestive disorders, deterioration of his general condition. A laboratory
analysis of feces intended for doctors revealed the presence of oval-shaped helminth eggs covered with a thick
bumpy shell. What is the diagnosis?
a) Fasciolosis d) Ascariasis
b) Trichocephalosis e) Diphyllobothriosis
c) Enterobiosis

292. Two children with pinworms have been identified in kindergarten. What kind of preventive measure should be
carried out to prevent infection of other children?
a) Cook meat and fish well d) Make vaccinations
b) No need for any activities e) Disinfect toys
c) Wash fruits and vegetables well

293.Name which of the helminthiasis listed below may be the cause of chronic appendicitis:
a) ascariasis, enterobiosis, trichocephalosis
b) trichinosis, hookworm, paragonimosis
c) vuhereriosis, trichinosis, hookworm
d) brugiosis, loaosis, opisthorchiasis
e) teniosis, trichocephalosis, fasciolosis

294.Which nematode larvae migrate through the human bloodstream during the development cycle?
a) Whipworms, intestinal eels, filaria
b) Ascaris, pinworms, crooked
c) Hookworms, trichinella, ascaris
d) Headed pinworms, necator, ascaris
e) Whipworms, hookworms, intestinal eels

295.During microscopy of a smear of feces of a schoolboy, yellow-brown eggs with a bumpy shell were revealed.
Which helminthiasis is it?
a) Pinworm d) Chainworm – Human Whipworm
b) Human Ascaris e) Broad Lent
c) Dwarf

296.A 10-year-old child complains of weakness, nausea, irritability. White helminths were found on length 5-10
mm. During microscopy of the scraping from the perianal folds, colorless eggs in the form of asymmetric ova were
revealed. What kind of helminth parasitizes a child?
a) Human Ascaris d) Trichinella
b) Duodenum Crooked head e) Whipworm
c) Pinworm

297.Echinococcosis is one of the most dangerous human helminthiases requiring surgical intervention. What method
is used for laboratory diagnosis of this disease?
a) X–ray d) Larvogelmintoscopy
b) Ovohelmintoscopy e) Biological samples
c) Immunological

298.During microscopy of the scraping from the perianal folds, colorless eggs were revealed in the child, having the
shape of asymmetric ovals, measuring 50×23 microns. What kind of helminthiasis is?
a) Ascaris d) Pinworm
b) Duodenum e) Dwarf chain
c) Human Whipworm

299. Primary hypovitaminosis occurs:


a) as a result of insufficient intake of vitamin(s) into the body or due to imbalance of nutrition
b) as a result of partial destruction of vitamins in the digestive tract
c) as a result of increased intake of proteins, fats and carbohydrates into the body
d) as a result of increased destruction of proteins, fats and carbohydrates in the body
e) as a result of increased intake of proteins and fats into the body

300.Water-soluble vitamins are include:


a) vitamins A and D D) vitamins B6
b) vitamins E and K e) vitamins C
c) vitamins PP

301. Find the characteristic symptoms of hypovitaminosis A.


a) Athe growth and development of the body slows c) Hypokeratosis.
down d) Delayed physical development
b) Body weight increases; e) Mental retardation

302.Define the most characteristic age for hypovitaminosis D.


a) young children; d) newborn children
b) conscripts; e) preschool children
c) teenagers 16-18 years old

303.Thiamine deficiency occurs when:


a) with a diet high in protein c) sleep disorders and depression
b) when eating a diet with a high proportion of d) decrease in blood pressure
refined carbohydrates e) when eating with a decrease in protein content

304. Symptoms of folic acid hypovitaminosis occur when:


a) the diet contains folic acid at a level of 50 mcg/day;
b) less than 5 mcg/day of folic acid enters the body with food for a long time;
c) it is impossible to determine the amount of folic acid entering the body.
d) lack of awareness of the population about the need to prevent folic acid deficiency
305. Hypovitaminosis E is considered a risk factor for:
a) Pathologies of pregnancy d) Damage to the skin
b) Diseases of the genitourinary system of boys e) Decrease in blood pressure
c) Atherosclerosis and angina pectoris
306. The toxic effect of calciferol (vitamin D) usually occurs when:
a) long-term increase in the diet of special vitamin preparations containing vitamin D in increased physiological
doses
b) the use in the diet of adulterated vegetable oil intended for animal feed and artificially enriched with vitamin D
c) diseases caused by vitamin D deficiency against the background of compensatory therapy
d) diseases caused by vitamin C deficiency against the background of compensatory therapy
e) use of products containing ascorbic acid in the diet

307. The toxic effect of vitamin C usually occurs when:


a) long-term increase in the diet of special vitamin preparations containing vitamin C
b) use of products containing ascorbic acid in the diet
c) diseases caused by vitamin C deficiency against the background of compensatory therapy
d) use of adulterated vegetable products in the diet
e) use of foods containing vitamins in the diet

308.6-year-old Denis was brought to the pediatrician by his parents with complaints of weakness, rapid fatigue, and
poor appetite. The parents also note that Denis often experiences sleep disturbances and mood changes. Upon objective
examination of the child, signs of exhaustion, dry skin, and brittle hair were found. Laboratory tests showed a low
level of iron in the blood, as well as low levels of vitamins D and B12. Which of the following methods is the most
effective for diagnosing hypovitaminosis in 6-year-old Denis?
a) Abdominal ultrasound
b) Computerized tomography (CT) of the brain
c) Urine analysis for protein content
d) Measurement of vitamin levels in the blood
e) Determination of growth and weight for age using specialized growth charts.

309.8-year-old Andrei was brought to the pediatrician by his parents with complaints of frequent episodes of fatigue,
reduced activity, and increased irritability. The parents also note that Andrei often experiences headaches and unusual
changes in his usual behavior. Objective examination revealed signs of muscular weakness, paleness of the skin, and
increased nail fragility. Laboratory tests showed a low level of iron in the blood, as well as a low level of vitamin
C.Which of the following methods is the most effective for diagnosing vitamin C deficiency in 8-year-old Andrei?
a) Prescribing oral vitamin supplements containing vitamin C
b) Intravenous administration of vitamin C preparations
c) Recommending increased consumption of fruits and vegetables rich in vitamin C
d) Applying skin patches with vitamin C
e)Using homeopathic remedies to increase the level of vitamin C in the body

310.7-year-old Anna was brought to the pediatrician by her parents with complaints of increased fatigue, sleep
disturbances, and irritability. Parents also note that Anna frequently experiences muscle and bone pain, especially
during prolonged activity. Upon objective examination, signs of muscle weakness and increased bone sensitivity were
identified. Laboratory tests revealed low levels of calcium and phosphorus in the blood, as well as a low level of
vitamin D.Which of the following treatment approaches is most suitable for 7-year-old Anna suspected of vitamin D
deficiency?
a) Prescribing oral vitamin complexes containing vitamin D, with a focus on calcium and magnesium.
b Intravenous administration of vitamin D preparations with regular monitoring of blood calcium levels.
c) Recommending increased consumption of fatty fish such as salmon and cod, rich in vitamin D.
d) Using specialized capsules with micronutrients and vitamins, including vitamin D, to raise their levels in the
body.
e) Conducting a course of extravagant therapy using vitamin injections, including vitamin D, in combination with
homeopathic remedies to boost immunity.

311. A pediatrician examined a two-month-old child. Mother's complaints of periodic anxiety, excessive sweating.
On examination: the back of the head is flattened, bald. The edges of the large fontanel are pliable. What kind of
disease are we talking about?
1. Vitamin D-resistant rickets 4. Rickets
2. Phosphate diabetes 5. Spasmophilia
3. Debre-de Toni-Fanconi syndrome

312. What is the most important sign of B12-deficiency anemia:


a) Hyperchromic anemia d) Anemia, lymphopenia, monocytopenia
b) Hypochromic anemia e) Anemia, reticulocytosis
c) Thrombocytopenia
313. The child, 3 months, child gets cow's milk. Mother complains of anxiety, sleep disturbances, increased sweating.
During the inspection: general condition was violated, attracts attention pallor, hair loss back, it is flat, asymmetrical,
a slight softening of the skull bones, the ribs are the rosary in a small amount. Organs and systems - within the age-
appropriate. What treatment should be?
a) Vitamin D in 400-600 IU per day
b) calcium in the age dose
c) Vitamin D 4000-10000 IU per day
d) Vitamin D 2000-4000 IU per day
e) Vitamin D 1000-2000 IU per day

314. The mother complains of lack of teeth and lower limb deformation in a child 8 months. The baby was born
weighing 3000 grams healthy from birth on artificial feeding, prevention of rickets received. During the inspection,
the overall condition is not violated, pale skin, teeth no large fontanel has a size of 3 x 3 cm lower aperture chest
expanded palpable ribs rosary. There O-curvature of the lower extremities. With the weakening of the heart listen
colors, functional systolic murmur. What is your diagnosis?
a) Rickets d) vitamin D-dependent rickets
b) congenital brittle bones e) Vitamin D-resistant rickets
c) Hondrodystrofy

315. A child 6.5 months, was born weighing 3500 g, body length 54 cm. Located breastfed. Mother complains of poor
appetite, anxiety child, pale skin and visible mucous membranes, recurrent abdominal cramps, liquid stool. Disturbing
symptoms listed above during the last 2 weeks against the backdrop of the introduction of the second breast feeding
in the form of porridge. The examination: heart rate - 136 in 1 min. Heart sounds loud, tachycardia, mild systolic
murmur over the top. Belly swollen, hard on palpation. The liver acts at 3 cm from the edge of costal arch. In the
analysis of blood: red blood cells - 2.8 * 10 12 degree / l, hemoglobin - 80 g / l, color index - 0.76. What was the cause
of anemia?
a) Malabsorption syndrome d) Breastfeeding
b) Diseases of the digestive tract inflammatory e) Protein-energy deficiency
c) Lack of iron in food

316. An 8-month-old infant presents with the primary complaint of irritability. He has been exclusively breastfed since
birth. His mother was not interested in providing any supplemental foods because her milk supply has been adequate.
Physical examination reveals a fussy infant who has frontal bossing and whose weight and height are both at the 25th
percentile. The infant becomes irritable with movement of the left arm. Arm radiography reveals a humeral fracture
and bowing of both radii. Chest radiography demonstrates enlargement of the costochondral junctions.
of the following, the most likely diagnosis is
a) Osteogenesisim perfecta d) Vitamin E deficiency
b) Vitamin D-deficient rickets e) Hondrodystrofy
c) Vitamin D-resistant rickets

317. Pneumonia during auscultation is characterized by:


a) The presence of dry wheezing, which increases on exhalation, wheezing breathing
b) The presence of crepitation or small-bubbly wet wheezing, weakened or bronchial breathing
c) The presence of dry wheezing, scattered throughout the pulmonary fields, hard breathing
d) The presence of small bubbly wheezes at the height of inspiration, various wet and dry wheezes, which change the
character after coughing
e) The presence of wet and dry wheezing

318. A 7-year-old child present with hypoalbuminemia, edema,hyperlipidemia and proteinuria.The edema is
in the periorbital region initially and eventually spreads to the rest of the body. The patient is given steroid
therapy and the disease goes away.What is a key morphological feature of the patient’s disease?
a. Fusion of the foot processes d. Hemosiderin laden macrophages in the kidney
b. Destruction of the basement membrane e. None of the above
c. Destruction of the glomerulus

319.Which of the following is the best next step after 8 weeks of proton pump inhibitor trial without improvement of
chronic epigastric discomfort?
1. Colonoscopy
2. H2 inhibitors
3. Continue proton pump inhibitors
4. Endoscopy with biopsy
5. Abdominal ultrasonography
320.What is an effective test for detection of H. pylori infection?
1. Ultrasound scan 4. Urine analysis
2. Urea breath 5. glomerular filtration rate (GFR)
3. Blood picture

321. Which of the following medical conditions is the most common cause of dyspepsia?
1. Functional dyspepsia 4. Pancreatitis
2. Gastric cancer 5. Esophageal cancer
3. Peptic ulcer disease

322. Which structure of Helicobacter pylori allows it to attach to the superficial gastric cells?
1. Cell wall 4. Flagella
2. Cell membrane 5. Pseudopodia
3. Cilia

323. Which of the following is the best next step in the management of Functional dyspepsia with alarm features?
a) Proton pump inhibitors d) Diet and lifestyle modification
b) Testing for Helicobacter pylori e) H2 antagonists
c) Endoscopy

324. Which of the following pathophysiological mechanisms is NOT responsible for injury in peptic ulcer disease?
1. Reflux of acid into the esophageal mucosa
2. Mucosal damage due to Helicobacter pylori and NSAIDs
3. Loss of blood supply to the mucosa
4. Increased acid production
5. Inhibition of prostaglandins

325. What types of medication are considered for a client with peptic ulcer disease (PUD)?
a) H2 receptor antagonists, antibiotics, proton pump inhibitors, probiotics
b) H2 receptor antagonists, antibiotics, proton pump inhibitors, mucosal protectants
c) H2 receptor antagonists, antibiotics, probiotics, antacids
d) H2 receptor antagonists, nonsteroidal anti-inflammatory drugs, proton pump inhibitors, mucosal protectants
e) Nonsteroidal anti-inflammatory drugs, antibiotics, proton pump inhibitors, mucosal protectants, probiotics

326. Which of the following is a side effect of omeprazole?


1. Increased risk of sepsis
2. Increased risk of developing hirsutism
3. Increased risk of enteric infection
4. Increased risk of anxiety
5. Increased risk of developing dermatitis nigricans

327. What is the mechanism of action of ranitidine?


1. Blocks histamine receptor type 2 in parietal cells
2. Blocks histamine receptor type 2 in the chief cells
3. Blocks histamine receptor type 1 in the surface mucous cells
4. Blocks hydrochloric acid production by the G cells
5. Blocks hydrochloric acid production by the gastric glands

328. A 17-year-old girls presents to the emergency department with complaints of pain in her abdomen that started
an hour after eating lunch. Which of the following is the diagnostic modality of choice in this patient?
1. CT scan with contrast 4. Nuclear medicine scan
2. CT scan without contrast 5. X-ray
3. Ultrasound

329. An obese teenager comes to the ER with episodes of vomiting and RUQ pain radiating to her right scapula.
Which of the following imaging modality would be most helpful to confirm acute cholecystitis?
1. CT abdomen 4. Plain x-ray of abdomen
2. Abdominal ultrasound 5. Esophagogastroduodenoscopy
3. MRI abdomen

330. A 17-year-old girl presents with right upper quadrant pain. She has a BMI of 32 kg/m2. She complains of
feeling nauseated at times. What is the probable type of gallbladder stone causing the right upper quadrant pain?
a) Pure cholesterol stone
b) Mixed stones
c) Combined stones
d) Pigment stone
e) Uric acid stone

331. What kind of supports can help infants manage viral bronchiolitis?
a. Antipyretics, nasal suction, hydration, supplemental oxygen
b. Antibiotics, nasal suction, hydration, stopping breastfeeding
c. Antipyretics, nasal suction, fluid restriction, supplemental oxygen
d. Antibiotics, nasal suction, hydration, supplemental oxygen
e. Antipyretics, stopping breastfeeding, supplemental oxygen

332. When a patient is diagnosed with acute bronchitis, which of the following investigations is usually performed
in order to rule out an accompanying pneumonia?
1. Chest X-ray
2. Bronchoscopy
3. HRCT-scan
4. Lung biopsy
5. MRI

333. 16-year-old male presents with a history of productive cough and low grade fever. He was previously healthy
and does not suffer from any chronic ailments. He is diagnosed with tracheobronchitis. What is the next step in
management?
a) Supportive care only
b) Inhaled steroids
c) Inhaled bronchodilators
d) Oral third generation cephalosporins
e) Oral neuraminidase inhibitors

334. Which factors would indicate a diagnosis of bronchitis?


a) History of acute onset of persistent cough for 1–3 weeks, no clinical signs of pneumonia (fever, rales,
tachypnea), an infiltrate on chest radiograph
b) History of acute onset of persistent cough for 1–3 weeks, no clinical signs of pneumonia (fever, rales,
tachypnea), chest X-ray to rule out pneumonia
c) History of acute onset of persistent cough for 1–3 weeks, no clinical signs of pneumonia (fever, rales,
tachypnea), a pleural effusion or empyema on chest radiograph
d) History of acute onset of persistent cough for 1–3 weeks, no clinical signs of pneumonia (fever, rales,
tachypnea), an elevated WBC count, in the range of 15,000-40,000/mm3
e) History of acute onset of persistent cough for 1–3 weeks, clinical signs of pneumonia (fever, rales, tachypnea),
positive blood culture

335. Which of the following is a common cause of acute pneumonia, especially in the pediatric population?
a) Mycoplasma pneumonia
b) Nocardia
c) Actinomycosis
d) Tuberculosis
e) Aspergillus

336. What age range of children should be treated with amoxicillin for bacterial pneumonia?
a. Children 5–8 years old
b. Children < 5 years old
c. Children 5–10 years old
d. Children > 5 years old
e. Adolescents

337. In patients with health care-associated pneumonia, which cephalosporin is initially included in the antibiotic
coverage regimen?
1. Ceftriaxone
2. Cefuroxime
3. Cefepime
4. Cefaclor
5. Cefazolin
338.Renal childhood diseases responsible for chronic hypertension include
a) hemolytic-uremic syndrome
b) acute tubular necrosis
c) congenital dysplastic kidney
d) pyelonephritis
e) renal trauma

339.Mother of a previously healthy 4 year old male complains of cough and wheeze. Boy had playing with a small
toy. During examination the right side of a chest show hyperresonance, diminished vocal resonance and poor air entry.
What is the most probable diagnosis?
a. Pneumonia
b. Asthma
c. Foreign body aspiration
d. Bronchitis
e. Bronchiolitis

340.Boy is 3 years old. Complaints: dyspnoe, fast tiredness, frequent episodes of respiratory diseases in history.
Borders of relative heart dullness are extended to the left, strengthening of the 2nd heart sound in the 2nd intercostal
interspace on the left side, hard systolo-dyastolic murmur in the second intercostal interspace on the left side and
above the clavicle (“machine noise”), which is conducted on interscapular interspace. What is the most probable
diagnosis?
a. Aortal stenosis
b. Patent arterial duct
c. Atrial septal defect
d. Ventricle septal defect
e. Isolated stenosis of arteria pulmonalis

You might also like